Sei sulla pagina 1di 30

MINISTRY OF PUBLIC HEALTH OF UKRAINE

Department of human resources policy, education and science

Testing Board

Student ID Last name

Variant ___________________

Test items for licensing examination

Krok 2
MEDICINE
General Instruction
Every one of these numbered questions or unfinished statements in
this chapter corresponds to answers or statements endings. Choose the
answer (finished statements) that fits best and fill in the circle with the
corresponding Latin letter on the answer sheet.

Authors of items: Ahafonova O.O., Ambrozevych Z.M., Barannyk S.I., Berezniuk V.V., Berezov V.M.,
Bilenko O.A., Bilyk O.V., Bilyk V.D., Blikhar V.Ye., Bolbot Yu.K., Borzova O.Yu., Bukhtieieva E.R.,
Bukhtiyarova O.H., Buriak V.M., Butina L.I., Butvyn I.M., Chaika O.O., Chekanov S.L., Chelpan L.L.,
Chonka I.I., Chuiko Yu.M., Dankyna I.A., Dashchuk A.M., Demchenko T.V., Desiatska Yu.V., Drin T.M.,
Duplenko P.Yu., Dzis N.P., Filippova O.Yu., Franchuk O.A., Hahara V.F., Havrylova L.O., Henyk N.I.,
Herasymenko O.I., Hovalenkova O.L., Hrydasova V.D., Hubka O.V., Hyrla Ya.V., Kalinovska I.V.,
Kaliy V.V., Kalyberdenko V.B., Kandyba V.P., Karliychuk O.O., Kirieieva T.V., Klymenko A.V.,
Koliush O.I., Kompaniyets K.M., Kondratenko P.H., Kondratiev V.O., Konopkina L.I., Konovalova N.V.,
Kopchak O.V., Korovka S.Ya., Kovalchuk P.Ye., Kovalenko S.V., Kovtunenko R.V., Krut Yu.Ya.,
Kryvenko Z.F., Kryvonosov M.V., Kryzhanovsky Yu.M., Kudria V.I., Lavrinchuk I.O., Lakusta N.M.,
Lebediuk M.M., Leshchenko K.A., Liulka O.M., Makieieva N.I., Malovany V.V., Marushko Yu.V.,
Martsynik Ye.M., Melnychuk L.V., Mierkulova N.F., Mitiunina N.I., Miziuk V.V., Moroz I.M.,
Muravska O.M., Murtazin L.M., Mysak A.I., Nadraha O.B., Nechytailo Yu.M., Nerianov K.Yu.,
Nikolaichuk O.M., Nishkumai O.I., Nykoniuk T.R., Olifierovska R.P., Parashchuk Yu.S., Pashchenko I.V.,
Pertseva N.O., Petrynych V.V., Pisotska L.A., Pohorelov O.V., Proniv L.M., Pryshliak O.Ya.,
Radchenko O.M., Reitmaier M.Y., Riapolova T.L., Romanenko V.N., Rudai V.V., Rudenko M.M.,
Rudenko S.M., Ruzhytska O.O., Samardakova H.O., Semeniak A.V., Semenukha K.V., Sennikov I.A.,
Serheta I.V., Shapkin V.Ye., Shkrobanets I.D., Shorikov Ye.I., Shusterman T.Y., Shvyhar L.V.,
Sidykh N.M., Sierkova V.K., Sikorska M.V., Smachylo I.V., Smoliak L.L., Snizhko S.S., Soboleva N.P.,
Sokolov O.B., Soldak I.I., Sonnyk H.T., Sorochan V.V., Sotnik Yu.P., Suk V.H., Svyrydova V.V.,
Svystunov I.V., Sylenko H.Ya., Sychova V.V., Synoverska O.B., Tarasova V.I., Teliushchenko O.D.,
Tiuieva N.V., Todoriko L.D., Tokariev A.V., Tonkohlas O.A., Tretiakevych Z.M., Troian V.I.,
Tykhonova S.A., Ukhal M.I., Vankhanen N.V., Vatanska I.Yu., Volianska A.H., Voloshyna L.O.,
Vorokhta Yu.M., Vykhovanets T.A., Yakovenko I.K., Yermachenko T.P., Yevtushenko V.V.,
Yurchenko I.V., Yutanova A.V., Zakharov V.K., Zheliba M.D., Zoria A.V. and Committees of professional
expertise

Item reviewers. Agafonova O.O., Anisimov Ye.M., Bab’yak T.Ye., Chursina T.Ya., Dyndar O.A.,
Grygorov Yu.B., Grynzovs’ky A.M., Gubka V.O., Gutsalenko O.O., Kalinina S.Yu., Karapetyan K.G.,
Khrapach V.V., Kolesnyk O.M., Kolosovych I.V., Kondratyuk V.Ye., Kopcha V.S., Kravchenko O.V.,
Kutovy O.B., Kuz’mina I.Yu., Malanchuk L.M., Martynyuk L.P., Mishchenko V.P., Muravs’ka O.M.,
Petrushenko V.V., Prokhorova M.P., Pryshlyak O.Ya., Puzanova O.G., Pyptyuk O.V., Shestakova I.V.,
Shevtsova T.I., Stovban I.V., Tsvirenko S.M., Usenko S.G., Vakal’uk I.P., Volyans’ka A.G.

The book includes test items for use at licensing integrated examination “Krok 2. Medicine” and
further use in teaching.

The book has been developed for students of medical, pediatric and medical-and-prophylactic
faculties and academic staff of higher medical educational establishments.

Approved by Ministry of Public Health of Ukraine as examination and teaching


publication based on expert conclusions (Orders of MPH of Ukraine of
14.08.1998 №251, of 27.12.1999 №303, of 16.10.2002 №374, of 29.05.2003 №233).

© Copyright Testing Board.


Крок 2 Medicine (англомовний варiант, iноземнi студенти) 2017 рiк 1

1. A 32-year-old welder complains of 5. 5 weeks after hypothermia a 22-year-


weakness and fever. His illness initially old patient developed fever, weakness,
presented as tonsillitis one month earlier. On muscle pain, inability to move independently.
examination: body temperature - 38,9o C , RR- Objectively: tenderness, induration of
24/min., HR- 100/min., BP- 100/70 mm Hg, shoulder and shin muscles, restricted active
hemorrhages on the legs, enlargement of the movements, erythema on the anterior surface
lymph nodes. Complete blood count: Hb- 70 of the chest. There is a periorbital edema
g/l, RBC- 2, 2 · 1012 /l, WBC- 3, 0 · 109 /l wi- with heliotropic erythema. Gottron’s sign is
th 32% of blasts, 1% of eosinophils, 3% of present. What investigation is required for the
band neutrophils, 36% of segments, 20% of diagnosis verification?
lymphocytes, and 8% of monocytes, ESR- 47
mm/hour. What is the cause of anemia? A. Muscle biopsy
B. Aminotransferase activity
A. Acute leukemia C. Pneumoarthrography
B. Chronic lympholeukemia D. ASO titer
C. Aplastic anema E. Rheumatoid factor
D. B12 -deficient anemia
E. Chronic hemolytic anemia 6. A 37-year-old woman complains of
headaches, nausea, vomiting, spasms. The
2. After a 5-day-long celebration of his onset of the disease occurred the day before
daughter’s wedding a 65-year-old patient due to her overexposure to cold. Objecti-
”saw” in his yard many cats, chickens, and vely: fever up to 40o C ; somnolence; rigid
rats. He tried to chase them away, but was neck; Kernig’s symptom is positive on the
scared off when the animals started to scold both sides; general hyperesthesia. Blood test:
him and tried to harm him. Make the di- leucocytosis, increased ESR. Cerebrospinal
agnosis: fluid is turbid, yellow-tinted. What changes of
the cerebrospinal fluid are most likely?
A. Delirium tremens
B. Senile psychosis A. Neutrophilic pleocytosis
C. Schizophrenia B. Lymphocytic pleocytosis
D. Organic brain syndrome C. Blood in the cerebrospinal fluid
E. Reactive hallucinosis D. Xanthochromia in the cerebrospinal fluid
E. Albuminocytological dissociation
3. An 8-year-old boy developed a temperature
of 37, 5o C two days after his recovery from 7. A 44-year-old patient complains of diffi-
the case of URTI. He complains of suffocati- cult urination, sensation of incomplete uri-
on, heart pain. Objectively: the skin is pale, nary bladder emptying. Sonographic exami-
tachycardia, the I heart sound is weakened, nation of the urinary bladder near the
short systolyc murmur in the 4th intercostal urethra entrance revealed an oval well-
area near the left edge of the breastbone. defined hyperechogenic formation 2x3 cm
What heart disorder such clincal presentati- large that was changing its position during the
on is characteristic of? examination. What conclusion can be made?
A. Nonrheumatic myocarditis A. Stone
B. Primary rheumatic carditis B. Malignant tumour of the urinary bladder
C. Myocardiodystrophy C. Urinary bladder polyp
D. Fallot’s tetrad D. Prostate adenoma
E. Cardiomyopathy E. Primary ureter tumour
4. A 28-year-old woman has been delivered 8. 4 days after a patient received a gunshot
to a hospital with acute pain in the lower wound to the soft tissues of middle third
abdomen. There was a brief syncope. The of the thigh, his condition suddenly began
delay of menstruation is 2 months. Objecti- deteriorating. There are complaints of bursti-
vely: the patient has pale skin, BP- 90/50 mm ng pain in the wound; pain has been increasi-
Hg, Ps- 110/min. Lower abdomen is extremely ng during the last 12 hours. Edema of skin
painful. Vaginal examination reveals uterus and hypodermic tissue quickly grows. Body
enlargement. Promtov’s sign (pain during bi- temperature is 38,2o C , heart rate is 102/min.
manual gynecological examination) is posi- The wound edges gape, are dull in color;
tive. Right uterine appendages are enlarged the muscles, viable as of day before, now
and very painful. Posterior vault hangs over. protrude into the wound, look boiled, are dull
What is the most likely diagnosis? in color, have dirty-gray coating, and fall apart
when held with forceps. What infection has
A. Right-sided tubal pregnancy developed in the wound?
B. Right ovary apoplexy
C. Acute right-sided salpingoophoritis
D. Pelvioperitonitis
E. Incipient abortion
Крок 2 Medicine (англомовний варiант, iноземнi студенти) 2017 рiк 2

A. Anaerobic A. Surgical intervention


B. Aerobic gram-negative B. Continuation of conservative therapy
C. Putrid C. Physiotherapy
D. Aerobic gram-positive D. Sanatorium-and-spa treatment
E. Diphtheria of the wound E. Physical training
9. A 35-year-old patient has been sufferi- 13. Caries morbidity rate is 89% among resi-
ng from an illness for 3 days. 5 days ago he dents of a community. It is determined that
returned from a trip to Africa. The onset of fluorine content in water is 0,1 mg/l. What
disease was accompanied by fever up to 40o C , preventive measures should be taken?
chills, acute headache, myalgia. In the axi-
llary region the lymph node enlarged up to A. Water fluorination
3x6 cm can be palpated. The lymph node is B. Tooth brushing
dense, intensely painful, slightly mobile, wi- C. Fluorine inhalations
thout clear margins; the skin over the node D. Sealant application
is hyperenic and tight. Tachycardia is present. E. Introduction of more vegetables to the diet
Make the preliminary diagnosis:
14. A patient received flame burns of both
A. Plague hands. On the dorsal and palmar surface of
B. Sepsis the hands there are blisters filled with serous
C. Tularemia fluid. The wrist joint region is hyperemic. The
D. Lymphadenitis forearms were not injured. What is the provi-
E. Anthrax sional diagnosis?

10. A parturient woman complains of pain in A. II degree flame burn of the hands, 4% of
her mammary gland. In the painful area there body surface area
is an infiltration 3x4 cm in size with softened B. II degree flame burn of the hands, 2% of
center. Body temperature is 38,5o C . What is body surface area
the most likely diagnosis? C. IIIa degree flame burn of the hands, 4% of
body surface area
A. Acute suppurative mastitis D. III degree flame burn of the hands, 4% of
B. Pneumonia body surface area
C. Pleurisy E. IIb degree flame burn of the hands, 2% of
D. Milk retention body surface area
E. Birth trauma
15. During the periodic medical examination
11. A 52-year-old patient, who has been an assembly fitter (works on soldering detai-
suffering from angina pectoris, for 2 weeks ls) didn’t report any health problems. Closer
has increasingly frequent pain attacks in the examination revealed signs of asthenic-
area behind his sternum and his need for ni- vegetative syndrome. Blood included red
troglycerine has increased. Objectively: the blood cells with basophilic aggregations
condition is of moderate severity. The skin is and a somewhat higher number of reti-
pale. Heart sounds are weakened, rhythmic. culocytes, urine had a high concentration of
Heart rate is 84/min. ECG shows no signs of delta-aminolevulinic acid. The complex of
focal myocardial injury. What is the most li- symptoms indicates the initial stage of chronic
kely diagnosis? intoxication with:
A. Progressive angina pectoris A. Lead
B. First-time angina pectoris B. Manganese
C. Stable NYHA functional class II angina C. Mercury
pectoris D. Tin
D. Variant angina pectoris E. Ethanol
E. Acute cardiac infarction
16. A 29-year-old woman came to a
12. A 9-year-old boy has been suffering gynecologist with complaints of irritabili-
from multiple bronchiectasis since he was 3 ty, tearfulness, headache, nausea, occasional
years old. Exacerbations occur frequently (3- vomiting, pain in the heart area, tachycardia
4 times a year), after conservative therapy attacks, memory impairment, meteorism.
there are short remission periods. The di- These signs appear 6 days before menstruati-
sease progresses, the child is physically on and disappear the day before menstruation
underdeveloped, presents with pale skin, or during its first 2 days. On vaginal exami-
acrocyanosis, deformed nail plates in the nation: the uterus and uterine appendages
shape of ”clock-face”. Bronchography reveals are without alterations. What diagnosis is the
saccular bronchiectases in the lower lobe of most likely?
the right lung. What further treatment tactics
should be chosen?
Крок 2 Medicine (англомовний варiант, iноземнi студенти) 2017 рiк 3

A. Premenstrual syndrome A. Blood creatinine


B. Algodismenorrhea B. Blood bilirubin
C. Ovarian apoplexy C. Blood sodium
D. Genital endometriosis D. Uric acid
E. Neurosis E. Fibrinogen
17. Examination of a group of persons living 21. A worker of a glass-blowing workshop
on the same territory revealed the following complains of headache, irritability, visual
common symptoms: dark-yellow pigmentati- impairment - he sees everything as if through
on of the tooth enamel, diffuse osteoporosis a ”net”. Objectively: hyperemic sclera, thi-
of bone apparatus, ossification of ligaments ckened cornea, decreased opacity of pupi-
and joints, functional disorders of the central ls, visual acuity is 0,8 in the left eye, 0,7 in
nervous system. This condition may be caused the right eye. The worker uses no means of
by the excessive concentration of the followi- personal protection. What is the most likely
ng microelement in food or drinking water: diagnosis?
A. Fluorine A. Cataract
B. Copper B. Conjunctivitis
C. Nickel C. Keratitis
D. Iodine D. Blepharospasm
E. Cesium E. Progressive myopia
18. An infant has been born at the 41st week 22. For a week a 42-year-old patient has been
of gestation. The pregnancy was complicated suffering from fever attacks followed by high
with severe gestosis of the second semester. temperature, which occur each 48 hours. Body
The weight of the baby is 2400 g, the height is temperature raises up to 40o C and decreases
50 cm. Objectively: the skin is flabby, the layer in 3-4 hours with excessive sweating. The pati-
of subcutaneous fat is thin, hypomyotonia ent presents with loss of appetite and general
is observed, neonatal reflexes are weak. fatigue. The skin is pale and sallow. The li-
The internal organs are without pathologic ver and spleen are enlarged and dense on
alterations. This newborn can be assessed as palpation. What method of diagnosis verifi-
a: cation would be most efficient?
A. Full-term infant with prenatal growth A. Microscopy of blood smear and thick blood
retardation film
B. Premature infant B. Complete blood count
C. Immature infant C. Bacteriological analysis
D. Postmature infant D. Immune-enzyme assay
E. Full-term infant with normal body weight E. Microscopy of hanging blood drop
19. A patient suffering from infiltrati- 23. A 28-year-old patient is a drug addict. He
ve pulmonary tuberculosis was prescribed has been sick for a year, when noticed general
streptomycin, rifampicin, isoniazid, pyrazi- weakness, increased sweating, and weight
namide, vitamin C . One month after the begi- loss. He often had cases of respiratory di-
nning of the treatment the patient started seases. Within the last 2 days he demonstrates
complaining of reduced hearing and tinnitus. intermittent fever with profuse night sweati-
What drug has such a side effect? ng, increased general weakness, developed di-
arrhea with mucus and blood admixtures. On
A. Streptomycin examination: polylymphadenopathy, herpetic
B. Isoniazid rashes in the oral cavity; on abdominal
C. Rifampicin palpation: the liver and spleen are enlarged.
D. Pyrazinamide What is the most likely diagnosis?
E. Vitamin C
A. HIV-infection
20. A 39-year-old man complains of morni- B. Herpetic stomatitis
ng headaches, appetite loss, nausea, morni- C. Chronic lymphatic leukemia
ng vomiting, periodic nasal hemorrhages. The D. Colon cancer
patient had a case of acute glomerulonephritis E. Chronic sepsis
at the age of 15. Examination revealed rise of
arterial pressure up to 220/130 mm Hg, skin 24. A 40-year-old patient has acute onset
hemorrhages on his arms and legs, pallor of of disease caused by overexposure to cold.
skin and mucous membranes. What biochemi- Temperature has increased up to 39o C .
cal parameter is the most important for maki- Foul-smelling sputum is expectorated duri-
ng diagnosis in this case? ng coughig. Various moist crackles can be
auscultated above the 3rd segment on the ri-
ght. Blood test: leukocytes - 15, 0 · 109 /l, stab
neutrophils - 12%, ESR- 52 mm/hour. On X-
ray: in the 3rd segment on the right there is a
Крок 2 Medicine (англомовний варiант, iноземнi студенти) 2017 рiк 4

focus of shadow 3 cm in diameter, low density, A. Risk of late abortion with hemorrhage
with fuzzy smooth margins and a clearing in B. Risk of late abortion without hemorrhage
its center. What disease is most likely in the C. The process of late abortion
given case? D. Incomplete late abortion
E. Attempted late abortion
A. Pneumonia complicated by an abscess
B. Infiltrative tuberculosis 29. A 65-year-old man was diagnosed with
C. Peripheral pulmonary cancer B12 -deficient anemia and the treatment was
D. Cystic echinococcosis prescribed. A week later control blood test
E. Pulmonary cyst was performed. What would be the early indi-
cator of the therapy effectiveness?
25. A 48-year-old patient was found to have
diffuse enlargement of the thyroid gland, A. Increased number of reticulocytes
exophthalmia, weight loss of 4 kg in 2 months, B. Increased hemoglobin level
sweating. Objectively: HR- 105/min, BP- C. Megaloblastic hematopoiesis
140/70 mm Hg. Defecation act is normal. D. Normoblastic hematopoiesis
What kind of therapy is recommended in this E. Increased erythrocyte number
case?
30. A 35-year-old woman complains of heart
A. Mercazolil pain (”aching and drilling”) occurring mai-
B. Radioactive iodine nly in the morning in autumn and spring and
C. Propranolol irradiating to the neck, back and abdomen;
D. Lugol’s solution rapid heartbeat; low vitality. Occurrence of
E. Thyroxine this condition is not associated with physical
activity. In the evening, the patient’s condition
26. A 26-year-old woman is suspected to improves. Study of somatic and neurological
suffer from systemic lupus erythematosus due status and ECG reveal no pathology. What
to systemic lesions of skin, vessels, joints, pathology is most likely to have caused these
serous tunics, and heart that developed after clinical presentations?
photosensitization. The following is detected:
LE cells, antibodies to native DNA, isolated A. Somatization depression
anti-centromere antibodies, rheumatoid B. Resting stenocardia
factor is 1:100, Wassermann reaction is posi- C. Pseudoneurotic schizophrenia
tive, circulating immune complex is 120 uni- D. Neurocirculatory asthenia
ts. What immunological indicators are consi- E. Hypochondriacal depression
dered to be specific to this disease?
31. On the 4th day after recovery from a
A. DNA antibodies cold a patient was hospitalized with complai-
B. Rheumatoid factor nts of solitary spittings of mucoid sputum.
C. Anti-centromere antibodies On the 2nd day there was a single discharge
D. Immunoglobulin A of about 250 ml of purulent blood-streaked
E. Increased circulating immune complex sputum. Objectively: the patient’s conditi-
on is moderately severe. Respiratory rate -
27. A woman came to a doctor with complai- 28-30/min., Ps- 96/min., BP- 110/70 mm Hg.
nts of increased body temperature up to Respiration over the left lung is vesicular,
37,8o C and moderately sore throat for the last over the right lung - weakened. There are
3 days. Objectively: mandibular lymph nodes various moist crackles over the lower lobe
are enlarged up to 3 cm. Palatine tonsils are and amphoric breath sounds near the angle of
hypertrophied, with gray coating that spreads scapula. What is the most likely diagnosis?
to the uvula and anterior pillars of the fauces.
What is the most likely diagnosis? A. Acute pulmonary abscess
B. Exudative pleuritis
A. Oropharyngeal diphtheria C. Acute focal pneumonia
B. Infectious mononucleosis D. Pleural empyema
C. Pseudomembranous (Vincent’s) tonsillitis E. Pyopneumothorax
D. Agranulocytosis
E. Oropharyngeal candidiasis 32. A 65-year-old woman on abdominal
palpation presents with a tumor in the umbi-
28. A primigravida at the term of 20 weeks lical region and above it; the tumor is 13x8 cm
complains of pain in her lower abdomen, in size, moderately painful, non-mobile, pulsi-
smearing blood-streaked discharge from the ng. On auscultation systolic murmur can be
genital tracts. Uterine tone is increased, fetus observed. What is the most likely diagnosis?
is mobile. On vaginal examination: the uterus
is enlarged according to the term, uterine A. Abdominal aneurysm
cervix is shortened to 0,5 cm, external cervical B. Gastric tumor
orifice is open by 2 cm. What is the most likely C. Arteriovenous aneurysm
diagnosis? D. Tricuspid insufficiency
E. Bicuspid insufficiency
Крок 2 Medicine (англомовний варiант, iноземнi студенти) 2017 рiк 5

33. A 45-year-old man complains of cough fi- 37. A patient with suspected pheochromocytoma
ts and tickling in his nasopharynx. He had has normal blood pressure in the periods
been staying for 10 days in the polluted area between the atacks and a tendency towards
created by the Chornobyl nuclear power plant tachycardia. Urine test revealed no pathologi-
accident. Rhinoscopy shows signs of severe es. It was decided to use a provocative test
nasopharynx irritation. What radionuclide is with histamine. What medication should be
the cause of this irritation? prepared to provide emergency care in case
of positive test result?
A. Radioactive iodine
B. Radioactive cesium A. Phentolamine
C. Radioactive strontium B. Pipolphen
D. Radioactive plutonium C. Nifedipine
E. Radioactive cobalt D. Mesatonum
E. Prednisolone
34. A 20-year-old patient complains of severe
headache, double vision, weakness, fever, 38. A 42-year-old woman complains of severe
irritability. Objectively: body temperature is pulsing headache in the frontoparietal area,
38,1o C , the patient is reluctant to contact, vertigo, palpitations. She has been sufferi-
sensitive to stimuli. There are ptosis of ng from hypertension for 3 years. Significant
the left eyelid, exotropia, anisocoria S>D, increase of BP occurs 2-3 times per month
pronounced meningeal syndrome. On lumbar and lasts for 3-8 hours. The left ventricle is
puncture the cerebrospinal fluid flowed out enlarged, heart sounds are clear, heart rate
under a pressure of 300 mm Hg, the fluid - 105/min., BP- 225/115 mm Hg. ECG: signs
is clear, slightly opalescent. 24 hours later of left ventricular hypertrophy. What drug
there appeared fibrin film. Protein - 1,4 g/l, would be the most effective for termination
lymphocytes - 600/3 per mm3 , sugar - 0,3 of cerebral crisis attack?
mmol/l. What is the provisional diagnosis?
A. Labetalol
A. Tuberculous meningitis B. Hydrochlorothiazide (Hypothiazide)
B. Meningococcal meningitis C. Captopril
C. Lymphocytic (Armstrong’s) meningitis D. Losartan
D. Syphilitic meningitis E. Clonidine (Clophelin)
E. Mumps meningitis
39. A 45-year-old patient complains of pain
35. A 37-year-old worker during a fire ended in the epigastric region, left subcostal area,
up in the area of high CO concentration. abdominal distension, diarrhea, loss of wei-
He was delivered to a hospital in unconsci- ght. He has been suffering from this condition
ous state. Objectively: the skin of his face for 5 years. Objectively: the tongue is moist
and hands is crimson. Respiration rate is with white coating near the root; deep palpati-
20/min. ECG: alterations specific for hypoxic on of abdomen reveals slight pain in the epi-
myocardium. Hourly diuresis is 40 ml. Blood gastric region and Мауо-Robson’s point. Li-
test: erythrocytes - 4, 5 · 1012 /l, Нb- 136 ver is painless and protrudes by 1 cm from the
g/l, color index - 0,89, ESR- 3 mm/hour, costal arch. Spleen cannot be palpated. What
carboxyhemoglobin - 5%. What criterion disease can be primarily suspected?
allows determining the severity of the pati- A. Chronic pancreatitis
ent’s condition? B. Atrophic gastritis
A. Carboxyhemoglobin concentration C. Peptic stomach ulcer
B. Respiratory disorders D. Chronic cholecystitis
C. ECG results E. Chronic enteritis
D. Extent of trophic disorders 40. For three years a 31-year-old woman has
E. Development of chronic renal failure been complaining of pain and swelling of
36. After a case of purulent otitis a 1-year-old her radiocarpal and metacarpophalangeal
boy has developed pain in the upper third of articulations, their reduced mobility in the
the left thigh, body temperature up to 39o C . morning, which persisted up to 1,5 hours.
Objectively: swelling of the thigh in its upper Two weeks ago she developed pain, swelli-
third and smoothed out inguinal fold. The li- ng, and reddening of her knee joints, her
mb is in semiflexed position. Active and passi- body temperature increased up to 37,5oC .
ve movements are impossible due to severe The treatment was belated. Examination of
pain. What diagnosis is the most likely? the internal organs revealed no pathologic
alterations. Diagnosis of rheumatoid arthritis
A. Acute hematogenous osteomyelitis was made. What alterations are the most li-
B. Acute coxitis kely to be visible on the arthrogram?
C. Intermuscular phlegmon
D. Osteosarcoma
E. Brodie’s abscess
Крок 2 Medicine (англомовний варiант, iноземнi студенти) 2017 рiк 6

A. Joint space narrowing, usuration resins. Objectively: lesion foci are symmetri-
B. Joint space narrowing, subchondral cally localized on both hands. Against the
osteosclerosis background of erythema with blurred margi-
C. Cysts in the subchondral bone ns there are papulae, vesicles, erosions, crusts,
D. Numerous marginal osteophytes and scales. What is the most likely pathology?
E. Epiphyseal osteolysis
A. Occupational eczema
41. A 58-year-old woman complains of B. Idiopathic eczema
spontaneous bruises, weakness, bleeding C. Allergic dermatitis
gums, dizziness. Objectively: the mucous D. Simple contact dermatitis
membranes and skin are pale with numerous E. Erythema multiforme
hemorrhages of various time of origin. Lymph
nodes are not enlarged. Ps- 100/min, BP- 45. A 10-year-old boy with symptoms of
110/70 mm Hg. There are no alterations of arthritis and myocarditis was delivered into
internal organs. Blood test results: RBC- a hospital. Based on clinical examination the
3, 0 · 1012 /l, Нb- 92 g/l, color index - 0,9, preliminary diagnosis of juvenile rheumatoid
anisocytosis, poikilocytosis, WBC- 10 · 109 /l, arthritis was made. What symptom is the most
eosinophils - 2%, stab neutrophils - 12%, contributive for the diagnostics of this di-
segmented neutrophils - 68%, lymphocytes sease?
- 11%, monocytes - 7%, ESR- 12 mm/h. What A. Reduced mobility of the joints in the
laboratory test should be performed additi- morning
onally to make the diagnosis? B. Regional hyperemia of the joints
A. Platelets C. Affection of the large joints
B. Reticulocytes D. Enlarged heart
C. Clotting time E. Increased heart rate
D. Osmotic resistance of erythrocytes 46. A 25-year-old patient was delivered to
E. Fibrinogen an infectious diseases unit on the 3rd day of
42. After tonsillectomy a woman with illness with complaints of headache, pain in
systemic lupus erythematosus, who has been the lumbar spine and gastrocnemius muscles,
taking prednisolone for a year, developed high fever, chills. Objectively: condition is of
acute weakness, nausea, vomiting, pain in the moderate severity. Scleras are icteric. Pharynx
right iliac area, watery stool up to 5 times is hyperemic. Tongue is dry with dry brown
per day. Ps- 96/min., BP- 80/50 mm Hg. What coating. Abdomen is distended. Liver is
preventive therapy should have been admini- enlarged by 2 cm. Spleen is not enlarged.
stered prior to the surgery? Palpation of muscles, especially gastrocnemi-
us muscles, is painful. Urine is dark in color.
A. Increase of prednisolone dosage Feces are normal in color. What is the most
B. Prescription of desoxycorticosterone likely diagnosis?
acetate
C. Prescription of antibiotics A. Leptospirosis
D. Administration of Ringer’s solution B. Viral hepatitis type A
E. Administration of 10% N aCl solution C. Malaria
D. Infectious mononucleosis
43. A 45-year-old woman complains of E. Yersiniosis
paroxysmal intolerable facial pain on the left
with attacks that last for 1-2 minutes. Attacks 47. A 28-year-old woman complains of
are provoked by chewing. The disease onset skin hemorrhages after minor traumas and
was two month ago after overexposure to spontaneous appearance of hemorrhages on
cold. Objectively: pain at the exit points of the the front of her torso and extremities. On
trigeminal nerve on the left. Touching near examination: the skin is variegated (old and
the wing of nose on the left induces new pain new hemorrhages), bleeding gums. Blood
attack with tonic spasm of the facial muscles. platelets - 20·109 /l; in the bone marrow there is
What is the most likely diagnosis among those increased number of megakaryocytes and no
listed? platelet production. Treatment with steroid
hormones was effective. What disease is it?
A. Trigeminal neuralgia
B. Glossopharyngeal neuralgia A. Idiopathic thrombocytopenic purpura
C. Temporomandibular joint arthritis B. Hemophilia
D. Facial migraine C. Rendu-Osler-Weber disease (Hereditary
E. Maxillary sinusitis hemorrhagic telangiectasia)
D. Disseminated intravascular coagulation
44. A 28-year-old man complains of skin E. Acute vascular purpura
rash and itching on the both of his hands.
The condition persists for 1,5 years. The 48. A 74-year-old patient was delivered into
exacerbation of his condition he ascribes to admission room with clinical presentations of
the occupational contact with formaldehyde acute deep vein thrombosis of the shin. What
Крок 2 Medicine (англомовний варiант, iноземнi студенти) 2017 рiк 7

symptom is the most typical of this pathology? A. Renal amyloidosis


B. Chronic glomerulonephritis
A. Homans’ sign C. Acute glomerulonephritis
B. Rovsing’s sign D. Heart failure
C. Courvoisier’s sign E. Chronic pyelonephritis
D. Mayo-Robson’s sign
E. Grey Turner’s sign 52. A 23-year-old man had taken 1 g of aspirin
to treat acute respiratory infection. After that
49. A 9-month-old child presents with fever, he developed an asthmatic fit with labored
cough, dyspnea. The symptoms appeared 5 expiration that was arrested by introduction
days ago after a contact with a person wi- of aminophylline. The patient has no medical
th URTI. Objectively: the child is in grave history of allergies. The patient has undergone
condition. Temperature is 38o C , cyanosis of two surgeries for nasal polyposis in the past.
nasolabial triangle is present. RR- 54/min, What is the most likely diagnosis?
nasal flaring during breathing is observed.
There was percussion dullness on the right A. Aspirin-induced asthma
below the scapula angle and tympanic sound B. Atopic bronchial asthma
over the other areas of lungs. Auscultati- C. Infectious allergic bronchial asthma
on revealed bilateral fine moist crackles D. Exercise-induced asthma
predominating on the right. What is the most E. Symptomatic bronchospasm
likely diagnosis?
53. A 50-year-old patient was delivered to
A. Pneumonia a hospital with complaints of blood traces
B. URTI in urine. Urination is painless and undi-
C. Acute laryngotracheitis sturbed. Macrohematuria had been observed
D. Acute bronchitis for 3 days. Objectively: kidneys cannot be
E. Acute bronchiolitis palpated, suprapubic area is without alterati-
ons, external genitalia are non-pathologic. On
50. The mother of a 3-month-old child came rectal investigation: prostate is not enlarged,
to a family doctor with complaints of her painless, has normal structure. Cystoscopy
child being physically underdeveloped and revealed no alterations. What is the most li-
suffering from cough attacks and dyspnea. kely diagnosis?
Anamnesis: the child is the result of the
second full-term pregnancy with the risk of A. Renal carcinoma
miscarriage (the first child died of pulmonary B. Bladder tuberculosis
pathology at the age of 4 months, according C. Varicocele
to the mother). Body mass at birth is 2500 D. Dystopic kidney
g. Cough attacks were observed from the fi- E. Necrotic papillitis
rst days of life, twice the child was treated 54. A man complains of constant dull pain
for bronchitis. Considering the severity of the in the perineum and suprapubic area, weak
child’s condition the doctor made the referral flow of urine, frequent difficult painful
for hospitalization. What diagnosis was most urination, nocturia. The patient has been
likely stated in the referral? suffering from this condition for several
A. Mucoviscidosis (Cystic fibrosis) months, during which urination was becomi-
B. Acute obstructive bronchitis ng increasingly difficult, and pain in the peri-
C. Recurrent obstructive bronchitis neum has developed. On rectal examinati-
D. Pertussis on: the prostate is enlarged (mainly its right
E. Acute obstructive pneumonia lobe), dense, asymmetrical, central fissure is
smoothed out, the right lobe is of stony densi-
51. A 46-year-old man notes swollen legs, ty, painless, tuberous. What disease is it?
weakness, sensation of fullness and heavi-
ness in the right subcostal area; it is the A. Prostate cancer
first occurrence of these signs in the pati- B. Prostate sclerosis
ent. The patient has 20-year-long history of C. Urolithiasis, prostatolith of the right lobe
rheumatoid arthritis. The liver and spleen D. Prostate tuberculosis
are enlarged and dense. Blood creatinine - E. Chronic congestion prostatitis
0,23 mmol/l, proteinemia - 68 g/l, cholesterol
- 4,2 mmol/l, urine specific gravity - 1012, 55. A boy was born at 32 weeks of gestation. 2
proteinuria - 3,3 g/l, isolated wax-like cyli- hours after the birth he developed respiratory
nders, leached erythrocytes in the vision field, distress (RD). The RD severity assessed by
leukocytes - 5-6 in the vision field. What is the Silverman score was 5. The respiratory di-
most likely complication? sorders progressed, respiratory failure could
not be eliminated by Martin-Bouyer CPAP
(continuous positive airway pressure). X-ray
of lungs shows reticular and nodular pattern,
air bronchogram. What is the most likely
cause of respiratory distress syndrome?
Крок 2 Medicine (англомовний варiант, iноземнi студенти) 2017 рiк 8

60. An infant is 2,5 months old. The onset of


A. Hyaline membrane disease the disease was gradual, the child had normal
B. Segmental atelectasis body temperature but presented with slight
C. Bronchopulmonary dysplasia cough. Within a week the cough intensified,
D. Congenital pulmonary emphysema especially at night; on the 12th day the child
E. Edematous hemorrhagic syndrome developed cough fits occurring up to 20 times
per day and followed by vomiting. There was
56. An 8-year-old child with a 3-year-long one instance of respiratory arrest. Make the
history of diabetes was hospitalized in diagnosis:
hyperglycemic coma. Specify the initial dose
of insulin to be administered: A. Pertussis
B. Parainfluenza
A. 0,1-0,2 U/kg of body weight per hour C. Congenital stridor
B. 0,05 U/kg of body weight per hour D. Respiratory syncytial infection
C. 0,2-0,3 U/kg of body weight per hour E. Adenovirus infection
D. 0,3-0,4 U/kg of body weight per hour
E. 0,4-0,5 U/kg of body weight per hour 61. Vaginal examination reveals the head of
the fetus, which fills the posterior surface of
57. A 17-year-old young man complains of symphysis pubis and hollow of the sacrum.
general weakness, trismus, twitching of the The lower edge of symphysis pubis, ischi-
muscles in his left shin. 7 days ago he pierced adic spines, and sacrococcygeal joint can be
his foot with a nail. Objectively: at the sole palpated. Where in the lesser pelvis is the fetal
of the foot there is a wound, 0,3х0,2 mm in head situated?
size, with small amount of serous-purulent
discharge, the skin around the wound is A. In the narrow plane of lesser pelvis
hyperemic. What is the most likely diagnosis? B. In the wide plane of lesser pelvis
C. Above pelvic inlet
A. Tetanus D. In the area of brim
B. Phlegmon E. In the area of pelvic outlet
C. Osteomyelitis
D. Infected wound 62. A man works in casting of nonferrous
E. Erysipelas metals and alloys for 12 years. In the air
of working area there was registered high
58. A patient with trauma of the lower thi- content of heavy metals, carbon monoxi-
rd of the forearm volar surface caused by a de, and nitrogen. During periodic health
glass shard came to a first-aid center. Objecti- examination the patient presents with
vely: flexion of the IV and V fingers is asthenovegetative syndrome, sharp pains
impaired, sensitivity of the inner dorsal and in the stomach, constipations, pain in the
palmar surfaces of the hand and IV finger is hepatic area. In urine: aminolevulinic acid
decreased. What nerve is damaged? and coproporphyrin are detected. In blood:
A. Ulnar reticulocytosis, low hemoglobin level. Such
B. Radial intoxication is caused by:
C. Median A. Lead and lead salts
D. Musculocutaneous B. Tin
E. Axillary C. Carbon monoxide
59. A 58-year-old patient complains of D. Nitric oxide
pain in the lower left extremity, which E. Zinc
aggravates during walking, sensation of cold 63. A 30-year-old multigravida has been
and numbness in the both feet. The patient in labour for 18 hours. 2 hours ago the
has been suffering from this condition for 6 pushing stage began. Fetal heart rate is
years. Objectively: the skin is pale and dry, clear, rhythmic, 136/min. Vaginal examinati-
with hyperkeratosis. On the left shin hair is on reveals complete cervical dilatation, the
scarce, ”furrow” symptom of inflamed ingui- fetal head in the pelvic outlet plane. Sagittal
nal lymph nodes is positive. Pulse cannot be suture is in line with obstetric conjugate, the
detected over the pedal and popliteal arteri- occipital fontanel is near the pubis. The pati-
es and is weakened over the femoral artery. ent has been diagnosed with primary uterine
In the right limb popliteal artery pulsation is inertia. What is the further tactics of labour
retained. What is the most likely diagnosis? management?
A. Arteriosclerosis obliterans of the lower A. Outlet forceps
extremities B. Labour stimulation
B. Obliterating endarteritis C. Cesarean section
C. Femoral artery thrombosis D. Skin-head Ivanov’s forceps
D. Raynaud’s disease E. Vacuum extraction of the fetus
E. Buerger’s disease (thromboangiitis obli-
terans) 64. A patient, who had eaten canned
Крок 2 Medicine (англомовний варiант, iноземнi студенти) 2017 рiк 9

mushrooms (honey agaric) three days A. Lactational mastitis


ago, developed vision impairment (di- B. Abscess
plopia, mydriasis), speech disorder, disturbed C. Erysipelas
swallowing. What type of food poisoning D. Dermatitis
occurred in the patient? E. Tumor

A. Botulism 69. A 3-year-old girl is being treated at a


B. Food toxicoinfection resuscitation unit with diagnosis ”acute ki-
C. Fusariotoxicosis dney failure, oligoanuric stage”. ECG: high T
D. Honey agaric poisonong wave, extended QRS complex, displacement
E. Lead salts poisoning of S-T interval downwards below the isoline.
What electrolyte imbalance is it?
65. An infant is 3 weeks old. Since birth
there has been observed periodical vomiti- A. Hyperkalemia
ng within a few minutes after feeding. The B. Hypokalemia
amount of vomitive masses does not exceed C. Hypocalcemia
the volume of previous feeding. The infant D. Hypercalcemia
has age-appropriate body weight. What is the E. Hyperphosphatemia
most likely cause of this symptom? 70. Mother of an 8-year-old girl complains
A. Pylorospasm that the child is too short and has excessi-
B. Esophageal chalasia ve body weight. Objectively: obesity with fat
C. Adrenogenital syndrome deposits on the torso and face (round moon-
D. Pyloristenosis like face), acne, striae on the thighs and lower
E. Esophageal achalasia abdomen, hirsutism. What hormone can cause
such symptoms, when in excess?
66. A newborn with gestational age of 31
weeks presents with hypotonia and depressed A. Cortisol
consciousness. Hematocrit is 35%, general B. Thyroxin
cerebrospinal fluid analysis shows increased C. Testosterone
content of erythrocytes and protein, and low D. Insulin
glucose. These data correspond with the clini- E. Glucagon
cal presentation of:
71. A 9-year-old girl complains of fever up to
A. Intracranial hemorrhage 37,5oC , headache, inertness, weakness, loss
B. Meningitis of appetite, stomachache, and frequent pai-
C. Sepsis nful urination. Provisional diagnosis of acute
D. Anemia pyelonephritis is made. Clinical urine analysis:
E. Prenatal infection specific gravity - 1018, no protein, leukocytes
- 10-15 in the vision field. What investigati-
67. From urine of a 14-year-old boy with on method can verify the diagnosis of urinary
the exacerbation of secondary obstructive system infection?
pyelonephritis Pseudomonas aeruginosa was
isolated with a titer of 1000000 microbes per A. Bacteriological inoculation of urine
1 ml. What antibiotic is the most advisable in B. Rehberg test (creatinine clearance test)
this case? C. Zymnytsky test (measurement of daily
diuresis)
A. Ciprofloxacin D. Complete blood count
B. Ampicillin E. Clinical urine analyses, dynamic testing
C. Cefazolinum
D. Azithromycin 72. During assessment of work conditions
E. Chloramphenicol at the mercury thermometer manufacture,
content of mercury vapors in the air of worki-
68. A 20-year-old woman on the 10th day ng area is revealed to exceed maximum
after her discharge from the maternity ward concentration limit. Specify the main way of
developed fever up to 39o C and pain in her mercury penetration into the body:
left mammary gland. On examination the
mammary gland is enlarged, in its upper outer A. Respiratory organs
quadrant there is a hyperemic area. In this B. Intact skin
area a dense spot with blurred margins can be C. Damaged skin
palpated. The patient presents with lactostasis D. Gastrointestinal tract
and no fluctuation. Lymph nodes in the ri- E. Mucous tunics
ght axillary crease are enlarged and painful.
Specify the correct diagnosis: 73. During health assessment of car drivers
and police officers on point duty, the physici-
ans detected carboxyhemoglobin in the blood
of the patients, weakened reflex responses,
disturbed activity of a number of enzymes.
Крок 2 Medicine (англомовний варiант, iноземнi студенти) 2017 рiк 10

Revealed professional health disorders are A. Sheehan’s syndrome (postpartum hypopi-


most likely to be associated with the effect of: tuitarism)
B. Ovarian amenorrhea
A. Carbon monoxide C. Turner’s syndrome
B. Sulfurous anhydride D. Ovarian exhaustion syndrome
C. Mental stress E. Galactorrhea-amenorrhea syndrome
D. Aromatic hydrocarbons
E. Nitric oxide 78. On laboratory investigation of a pork
sample there is 1 dead trichinella detected
74. On the 3rd day of life a newborn, in 24 sections. This meat should be:
who had suffered birth asphyxia, developed
hemorrhage from the umbilical wound. A. Handed over for technical disposal
Laboratory analysis reveals hypocoagulati- B. Allowed for sale with no restrictions
on, thrombocytopenia, and hypothrombi- C. Processed and sold through public catering
nemia. What is the cause of such clinical network
developments? D. Processed for boiled sausage production
E. Frozen until the temperature of - 10o C is
A. Disseminated intravascular coagulation reached in the deep layers, with subsequent
B. Hemorrhagic disease of newborn exposure to cold for 15 days
C. Congenital angiopathy
D. Thrombocytopenic purpura 79. Examination of a 43-year-old man
E. Umbilical vessel trauma objectively revealed pallor of skin and
mucous membranes, loss of tongue papillae,
75. A 46-year-old woman came to a materni- transverse striation of fingernails, cracks in
ty clinic with complaints of moderate blood the mouth corners, tachycardia. Blood test
discharge from the vagina, which developed results: Hb- 90 g/l, anisocytosis, poikilocytosis.
after the menstruation delay of 1,5 months. The most likely causative factor of this condi-
On vaginal examination: the cervix is clean; tion is the inadequate intake of:
the uterus is not enlarged, mobile, painless;
appendages without changes. Make the di- A. Iron
agnosis: B. Copper
C. Zinc
A. Dysfunctional uterine bleeding D. Magnesium
B. Adenomyosis E. Selene
C. Ectopic pregnancy
D. Submucous uterine myoma 80. A 52-year-old woman presents with
E. Cancer of the uterine body affected mucosa in the mouth angles where
fissures, erosions, and ulcers develop; verti-
76. A patient has the second and third degree cal fissures appear on the lips during their
burns of the 15% of the body surface. On the closing (cheilosis); there are tongue alterati-
20th day after the trauma the patient presents ons (glossitis), angular stomatitis, seborrheic
with sharp increase of body temperature, dermatitis around the mouth and wings of
general weakness, rapid vesicular respiration; the nose, and pericorneal injection. The listed
facial features are sharpened, BP is 90/50 mm symptoms are characteristic of:
Hg, heart rate is 112/min. What complication
is it? A. B2 -hypovitaminosis
B. B1 -hypovitaminosis
A. Sepsis C. C -hypovitaminosis
B. Pneumonia D. P P -hypovitaminosis
C. Acute intoxication E. A-hypovitaminosis
D. Purulent bronchitis
E. Anaerobic infection 81. Two days ago a woman fell from the hei-
ght of 1,5 m. She complains of severe thoracic
77. A 30-year-old woman complains of pain on the left and dyspnea. Chest X-ray
amenorrhea that lasts for 2 years after she reveals hydropneumothorax on the left with
has given birth, loss of hair and body weight. fluid level at the 7th rib and the lung collapsed
The labor was complicated with hemorrhage by 1/3. The 6th-7th ribs are fractured along
caused by uterine hypotonia. Objectively the the scapular line. Serohemorrhagic fluid was
patient is of asthenic type, her external geni- obtained during thoracic puncture. What
talia are hypoplastic, the uterine body is small treatment tactics should be chosen?
in size and painless. No uterine appendages
can be detected. What is the most likely di-
agnosis?
Крок 2 Medicine (англомовний варiант, iноземнi студенти) 2017 рiк 11

A. Thoracocentesis on the left at the 7th shoulders, the head is bowed forward, the
intercostal space thorax is flattened, the stomach is bulging.
B. Thoracocentesis at the 2nd intercostal space In the vertebral column there are deepened
C. Repeated puncture of the pleural cavity cervical and lumbar flexures. What posture
D. Intubation of trachea with artificial does the child have?
pulmonary ventilation
E. Alcohol-novocaine block of the fracture A. Kyphosis
area B. Lordosis
C. Stooping
82. A patient complains of suppuration from D. Corrected
the ear and impaired hearing of the left ear, E. Normal
which have been observed for the past 6
years. The patient had periodical headaches, 86. A 46-year-old woman has been hospitali-
general indisposition, fever. Objectively: zed with open fracture of the left thigh in
otoscopy of the external auditory meatus its middle third. She underwent the surgery
revealed mucopurulent odorless substance. - fixation with extraosseous osteosynthesis
The eardrum is of normal color, with central plates. On the 4th day after the surgery
perforation. What is the most likely di- she developed pain in the wound, body
agnosis? temperature rose over 39o C . What measures
should be taken in this case?
A. Chronic mesotympanitis
B. Otosclerosis A. Undo the sutures, drain the wound, and
C. Acute otitis media prescribe antibiotics
D. Chronic epitympanitis B. Prescribe broad spectrum antibiotics and
E. Chronic sensorineural hearing loss hormonal agents
C. Administer antibiotics intraosseously and
83. In 2 hours after a traffic accident a 28-year- hypothermia locally
old man in grave condition was delivered to a D. Inject antibiotics into the area surroundi-
hospital. The patient complains of abdominal ng the wound, prescribe spasmolytics and
pain. He received a blow to the abdomen wi- analgesics
th the steering wheel. Objective examination E. Remove the fixation, prescribe sulfanilami-
revealed the following: the abdomen does not des
participate in respiration, is tense and acutely
painful on palpation; the abdominal muscles 87. A multigravida on the 38th week of her
are defensively tense, peritoneal irritation si- pregnancy complains of increased BP up to
gns are positive, hepatic dullness is absent. BP 140/90 mm Hg, edema of the shins for 2 weeks.
is 90/60 mm Hg, heart rate is 120/min. What In the last month she gained 3,5 kg of weight.
further treatment tactics should be chosen? Urine analysis: protein - 0,033 g/l. Make the
diagnosis:
A. Laparotomy
B. Laparoscopy A. Mild preeclampsia
C. Cold to the abdomen B. Moderate preeclampsia
D. Ultrasound investigation C. Pregnancy hypertension
E. Laparocentesis D. Severe preeclampsia
E. Pregnancy edema
84. A 28-year-old woman complains of
increased intervals between menstruations, 88. A 44-year-old patient with postinfarcti-
up to 2 months, and hirsutism. Gynecological on cardiosclerosis presents with frequent
examination revealed the following: ovaries heart rate disorders and lower extremity
are enlarged, painless, and dense; no alterati- edema. Objectively: Ps- 95/min., irregular,
ons of the uterus. US of the lesser pelvis: 10-12 extrasystoles per minute. BP- 135/90
ovaries are 4-5 cm in diameter, with numerous mm Hg. The 1st heart sound at the apex is
enlarged follicles on the periphery. X-ray of weakened. Pulmonary respiration is rough.
the skull base: sellar region is widened. What The liver is enlarged +2 cm. ECG: irregular
is the most likely diagnosis? sinus rhythm, heart rate - 95/min, frequent
polytopic ventricular extrasystoles. What anti-
A. Stein-Leventhal syndrome (polycystic arrhythmic drug is advisable in this case for
ovarian syndrome) treatment and prevention of extrasystole?
B. Algodismenorrhea
C. Sheehan syndrome (postpartum hypopitui- A. Amiodarone
tarism) B. Lidocaine
D. Premenstrual syndrome C. Mexiletine
E. Morgagni-Stewart-Morel syndrome D. Quinidine
(metabolic craniopathy) E. Novocainamide (Procainamide)

85. Posture of an 11-year-old boy was determi- 89. A 60 year-old woman has been suffering
ned during preventive examination. The chi- from weakness, dizziness, and fatigue over
ld presents with curled forward rounded the last year. Recently she has also developed
Крок 2 Medicine (англомовний варiант, iноземнi студенти) 2017 рiк 12

dyspnea, paresthesia. Objectively: the skin A. Hepatocellular carcinoma of the liver


and mucous membranes are pale and slightly B. Hepatocellular insufficiency
icteric. The tongue is smooth due to the loss of C. Hypersplenism
lingual papillae. Liver and spleen are located D. Cholestasis
at the costal margin. Blood count: Hb- 70 g/l, E. Congestive heart failure
RBC- 1, 7·1012 /l, color index - 1,2, macrocytes. 93. A 43-year-old man has undergone a
Administer the patient a pathogenetically surgery for osteomyelitis of the left thigh.
justified drug: On the 6th day the patient’s condition was
A. Vitamin B12 complicated with sepsis. Despite complex
B. Vitamin B6 therapy of sepsis on the 9th day the patient
C. Ascorbic acid persistently presents with temperature up to
D. Iron preparations 40o C , heart rate is 110/min., respiration rate
E. Vitamin B1 is 23/min., BP is 100/60 mm Hg. Blood test:
leukocytes - 16 · 109 /l, band neutrophils - 16%.
90. After excessive consumption of fatty food What phase of clinical course is it?
a 60-year-old woman suddenly developed
pain in her right subcostal area, nausea, bi- A. Catabolic
le vomiting, sharp bitter taste in her mouth. B. Anabolic
In 2 days she developed jaundice, her uri- C. Rehabilitation
ne darkened. Objectively: sclera and skin are D. Functional
icteric, the abdomen is distended, the liver is E. Tension
enlarged by 3 cm, soft and painful on palpati-
on, Ortner’s, Murphy’s, Kehr’s, Zakharyin’s, 94. A 23-year-old woman presents with di-
Mayo-Robson’s signs are positive. What di- abetes mellitus type 1. She complains of
agnostic technique should be used in the first weakness, headache, nausea, and vomiting.
place? Objectively: temperature is 37,6o C , heart rate
is 98/min., BP is 95/65 mm Hg, respiration rate
A. US of the gallbladder and bile duct is 32/min., loud. Smell of acetone is detected,
B. Fibrogastroduodenoscopy heart sounds are muffled, pulse is rhythmic.
C. X-ray of the abdomen The stomach is sensitive in the epigastrium.
D. Radionuclide scanning of the liver and Costovertebral angle tenderness (Murphy’s
gallbladder punch sign) is present. Blood glucose is 28,5
E. Laparoscopy mmol/l; blood leukocytes - 16, 5 · 109 /l. In uri-
ne: acetone ++, leukocytes - 25-40 in the vision
91. A 42-year-old woman working at a poultry field. Blood pH is 7,1. What correction method
farm complains of dyspnea, thoracic pain on would be the most advisable?
the left, increased body temperature up to 38-
39o C in the evening, and cough. The patient A. Infusion of normal saline + insulinotherapy
suffers from essential hypertension. Objecti- in small portions
vely: vesicular respiration in the lungs, vesi- B. Infusion of 5% glucose + insulinotherapy in
cular resonance without alterations. X-ray large portions
of both lungs reveals numerous small low- C. Infusion of 2,5% sodium bicarbonate +
intensity foci, 2-3 mm in size, located in a row insulinotherapy
along the blood vessels. ESR- 32 mm/hour. D. Intravenous administration of antibiotics +
What is the most likely diagnosis? insulinotherapy
E. Infusion of dextran solutions + insuli-
A. Acute disseminated tuberculosis notherapy
B. Infiltrative tuberculosis
C. Pulmonary carcinomatosis 95. A 57-year-old woman complains of a
D. Focal tuberculosis sensation of esophageal compression, palpi-
E. Community-acquired pneumonia tations, difficult breathing when eating solid
food, occasional vomiting with a full mouth,
92. A 65-year-old patient has been suffering ”wet pillow” sign at night for the last 6
from liver cirrhosis associated with hepati- months. Objectively: body tempearture -
tis C virus for 7 years. During the last 3 39o C , height - 168 cm, weight - 72 kg, Ps-
weeks the patient developed severe edema 76/min, BP- 120/80 mm Hg. X-ray study
of the lower extremities, the abdomen is si- revealed considerable dilation of esophagus
gnificantly distended with fluid. Ultrasound: and its constriction in the cardial part. What
signs of liver cirrhosis, portal hypertensi- pathology is the most likely to have caused
on, lymph nodes are in the area of hepatic dysphagia in this patient?
portal. Concentration of α-fetoprotein in
blood serum is 285 ng/ml. What complicati- A. Achalasia cardiae
on could have developed in this case? B. Primary esophageal spasm
C. Hiatal hernia
D. Esophageal carcinoma
E. Reflux esophagitis
Крок 2 Medicine (англомовний варiант, iноземнi студенти) 2017 рiк 13

96. A 47-year-old man is employed at the neck veins, peripheral pulse is absent, the
weaving workshop, has 15-year-long record carotid artery pulse is rhythmic, 130/min., BP
of service at this factory; his work conditi- is 60/20 mm Hg. Auscultation of the heart
ons are associated with high-frequency and reveals extremely muffled sounds, percussion
high-intensity noise. During periodical exami- reveals heart border extension in both directi-
nation he was diagnosed with occupational ons. What is the optimal treatment tactics for
deafness. What are the grounds for making this patient?
such a diagnosis?
A. Pericardiocentesis and immediate
A. Audiometry data and hygienic assessment thoracotomy
of working environment B. Oxygen inhalation
B. Record of service at this factory C. Puncture of the pleural cavity on the left
C. Noise characteristic at this factory D. Conservative treatment, infusion of
D. Central nervous system examination results adrenomimetics
E. Inner ear examination results E. Pleural cavity drainage
97. A 37-year-old woman complains of acute 101. A patient with chronic pancreatitis
pain in the genital area, swelling of the complains of diarrhea occurring up to 5 ti-
labia, pain when walking. Objectively: body mes per day (no blood traces), loss of body
temperature is 38,7o C , Ps- 98/min. In the weight, abdominal distention, dryness of skin,
interior of the right labia there is a dense, loss of hair, thirst, bleeding gums, convulsi-
painful tumor-like formation 5,0x4,5 cm in si- ons. Complete blood count: leukocytes -
ze, the skin and mucous membrane of genitals 5, 8 · 109 /l; Hb- 86 g/l; ESR- 15 mm/g; Blood
are hyperemic, there is profuse foul-smelling protein test: protein - 48 g/l; albumins - 28
discharge. What is the most likely diagnosis? g/l. What indicators of coprological analysis
would accompany this syndrom?
A. Acute bartholinitis
B. Labial furuncle A. Steatorrhea, creatorrhea
C. Acute vulvovaginitis B. Large amount of mucus, amylorrhea
D. Bartholin gland cyst C. Large amount of starch grains and cellulose
E. Carcinoma of vulva D. Gas bubbles, acid reaction
E. Large numbers of iodinophilous microbes
98. A woman undergoing in-patient treatment
for viral hepatitis type B developed headache, 102. A 35-year-old patient complains of
nausea, recurrent vomiting, memory lapses, heartburn, gasseous and sour eructation,
flapping tremor of her hands, rapid pulse. burning constricting pain behind the sternum
Sweet smell from the mouth is detected. Body and along the esophagus, developing when
temperature is 37,6o C , heart rate is 89/min. bowing the torso to the front. No previ-
What complication developed in the patient? ous examination; the patient takes almagel
at his own discretion, after which he notes
A. Acute liver failure improvement of his general state. Make the
B. Ischemic stroke provisional diagnosis:
C. Gastrointestinal hemorrhage
D. Hypoglycemic shock A. Gastroesophageal reflux disease
E. Meningoencephalitis B. Functional dyspepsia
C. Cardiospasm
99. A 42-year-old man was delivered to a D. Ulcer disease of the stomach
surgical in-patient department with complai- E. Ulcer disease of the duodenum
nts of icteric skin, pain in the right subcostal
area. Biochemical blood analysis: total bili- 103. The left hand of a newborn is extended
rubin - 140 mcmol/l, direct bilirubin - 112 in all its joints, stretched along the torso, and
mcmol/l. On US: choledoch duct - 1,4 cm, pronated in the forearm. Active movements
a concrement is detected in the distal area. of the shoulder joint are retained. The hand
Gallbladder is 40 cm, no concrements. What is flattened, atrophied, cold to touch, hangs
treatment tactics should be chosen? passively. Grasping and Babkin’s reflexes are
absent at the affected side. Hemogram indi-
A. Endoscopic papillosphincterotomy cators are normal. Make the most likely di-
B. Laparoscopic cholecystectomy agnosis:
C. Laparotomy with choledoch duct drain
D. Laparotomy with cholecystectomy A. Inferior distal obstetrical paralysis
E. Threatment in an infectious diseases hospi- B. Osteomyelitis
tal C. Proximal obstetrical paralysis
D. Complete obstetrical paralysis
100. 4 weeks after myocardial infarction a 56- E. Hypoxic-ischemic encephalopathy
year-old patient developed acute heart pain,
marked dyspnea. Objectively: the patient’s 104. A 34-year-old man is being treated for
condition is extremely grave, there is marked schizophrenia exacerbation in a psychiatric
cyanosis of face, swelling and throbbing of unit. Objectively: the patient remains in bed,
Крок 2 Medicine (англомовний варiант, iноземнi студенти) 2017 рiк 14

is sluggishly mobile, unresponsive, does not Objectively: the patient is exhausted, there
react to questions. His position is unvari- are enlarged supraclavicular lymph nodes.
ed, hypomimic, snout reflex and Dupre’s Esophagoscopy revealed no esophageal
symptom are present, muscles exhibit waxy pathology. Which of the following investigati-
flexibility. He has been remaining in this ons is the most appropriate in this case?
state for approximately a week. Feeding is
parenteral. Determine the neuromotor di- A. Computed tomography of chest and medi-
sturbance: astinum
B. X-ray of lungs
A. Catatonic stupor C. Multiplanar imaging of esophagus
B. Depressive stupor D. Radioisotope investigation of chest
C. Psychogenic stupor E. Ultrasound investigation of mediastinum
D. Anergic stupor
E. Exogenic stupor 109. A 15-year-old girl complains of dizzi-
ness and sensation of lack of air that she
105. A 14-year-old girl has been delivered to develops in emotionally straining situations.
a gynecological department with complaints Relief occurs after she takes corvalol. Objecti-
of profuse blood discharge from her genital vely: hyperhidrosis and marble-like pattern
tract for 2 weeks. Anamnesis: menstruation si- of the skin of her palms and feet. Clinical
nce 13, irregular, painful, profuse; the last one and instrumental examination revealed no
was 2 months ago. Objectively: pale skin and organic alterations of the central nervous,
mucosa, BP- 100/60 mm Hg, Hb- 108 g/l. The cardiovascular, and respiratory systems. What
abdomen is soft and painless on palpation. provisional diagnosis can be made?
Rectal examination revealed no pathologies
of reproductive organs. What condition is it? A. Somatoform autonomic dysfunction
B. Obstructive bronchitis
A. Juvenile uterine hemorrhage (Dysfuncti- C. Bronchial asthma
onal) D. Stenosing laryngotracheitis
B. Hypomenstrual syndrome E. Acute epiglottitis
C. Inflammation of uterine appendages (Pelvic
110. A 14-year-old patient complains of
inflammatory disease) alopecia foci on his scalp. The patient has
D. Pelviperitonitis been presenting with this condition for 2
E. Endometritis weeks. Objectively: on the scalp there are
106. An infant cries during urination, the several small oval foci with blurred margins.
foreskin swells and urine is excreted in The skin in the foci is pink-red, the hairs are
drops. What approach to treatment should broken off at 4-5 mm length or at skin level.
be chosen? Under Wood’s lamp there are no foci of green
luminescence detected. What disease is it?
A. Create an opening into the foreskin cavity
B. Prescription of α-adrenergic blocking A. Trichophytosis capitis
agents B. Syphilitic alopecia
C. Prescription of antispasmodic agents C. Alopecia areata
D. Urinary bladder catheterization D. Scleroderma
E. Epicystostomy E. Psoriasis

107. A 20-year-old patient complains of pain 111. A 38-year-old patient has been delivered
in the left lumbar region, arterial pressure rise by an ambulance to a surgical department
up to 160/110 mm Hg. US revealed that the with complaints of general weakness, indi-
structure and size of the right kidney were wi- sposition, black stool. On examination the
thin age norms, there were signs of the 3rd patient is pale, there are dotted hemorrhages
degree hydronephrotic transformation of the on the skin of his torso and extremiti-
left kidney. Doppler examination revealed es. On digital investigation there are black
an additional artery running to the lower feces on the glove. Blood test: Hb- 108 g/l,
pole of the kidney. Excretory urogram shows thrombocytopenia. Anamnesis states that si-
a narrowing in the region of ureteropelvic milar condition was observed 1 year ago.
junction. Specify the treatment tactics: Make the diagnosis:

A. Surgical intervention A. Thrombocytopenic purpura


B. Administration of spasmolytics B. Hemophilia
C. Administration of ACE inhibitors C. Ulcerative bleeding
D. Kidney catheterization D. Rectal tumor
E. Administration of β -blockers E. Nonspecific ulcerative colitis

108. A 49-year-old patient consulted a doctor 112. A 35-year-old patient developed an epi-
about difficult swallowing, voice hoarseness, leptic attack with tonoclonic spasms that
weight loss. These symptoms have been lasted for 3 minutes. After the attack the pati-
gradually progressing for the last 3 months. ent fell asleep but in 5 minutes the second
Крок 2 Medicine (англомовний варiант, iноземнi студенти) 2017 рiк 15

attack occurred. The first step of emergency 22. His current condition has changed acutely:
aid would be to: for 3 days the patient has been refusing
to leave his home. He claims that there is
A. Ensure patency of airways a ”telepathy” occurring between him and
B. Take blood from the vein for analysis the other people, through which he receives
C. Introduce diazepam intravenously ”thoughts of strangers” and transmits his own
D. Prescribe antiepileptic drugs thoughts for everyone to hear. He is convi-
E. Administer chloral hydrate via an enema nced that his thoughts and actions are being
manipulated through this ”telepathy”. Make
113. A 23-year-old woman has been suffering the preliminary diagnosis:
from a mental disease since the age of 18,
the course of disease has no remission peri- A. Paranoid schizophrenia
ods. At a hospital the patient mostly presents B. Depressive episode
with non-purposeful foolish excitation: she C. Manic episode
makes stereotypic grimaces, exposes herself, D. Organic delirium
publicly masturbates with a loud laughter, E. Acute reaction to stress
repeates stereotypical abusive shouts. The
patient should be prescribed: 118. A 35-year-old woman has gained 20 kg
weight within a year with the normal diet.
A. Neuroleptics She complains of chills, sleepiness, shortness
B. Antidepressants of breath. The patient’s mother and sister are
C. Tranquilizers corpulent. Objectively: height - 160 cm, wei-
D. Nootropics ght - 92 kg, BMI- 35,9. Obesity is uniform,
E. Mood stabilizers there are no striae. The face is amimic. The
skin is dry. The tongue is thickened. Heart
114. A young woman suffering from sounds are muffled. HR- 56/min, BP- 140/100
seborrhea oleosa has numerous light-brown mm Hg. The patient has been suffering from
and white spots on the skin of her torso amenorrhea for 5 months, has constipations.
and shoulders. The spots have clear margi- TSH- 28 mcIU/l (norm is 0,32-5). Craniogram
ns, branny desquamation, no itching. What shows no pathology. What is the etiology of
provisional diagnosis can be made? obesity?
A. Pityriasis versicolor A. Hypothyroid
B. Torso dermatophytosis B. Hypo-ovarian
C. Seborrheic dermatitis C. Hypothalamic-pituitary
D. Pityriasis rosea D. Alimentary and constitutive
E. Vitiligo E. Hypercorticoid
115. 10 days after birth a newborn developed 119. A patient complains of painless ”sores”
sudden fever up to 38,1o C . Objectively: the on his penis and inguinal lymph nodes
skin in the region of navel, abdomen and enlargement. Synthomycin emulsion that the
chest is erythematous; there are multiple pea- patient have been applying to the ”sores” was
sized blisters with no infiltration at the base; ineffective. Objectively: on the inner leaf of
isolated bright red moist erosions with epi- the foreskin there are three closely situated
dermal fragments are observed on the peri- rounded erosions, 0,5 cm in diameter, with
phery. What is the provisional diagnosis? dense infiltration that can be palpated at their
A. Epidemic pemphigus of newborn bases. Make the preliminary diagnosis:
B. Syphilitic pemphigus A. Primary syphilis
C. Streptococcal impetigo
D. Vulgar impetigo B. Herpes simplex (Herpes pro genitalis)
E. Atopic dermatitis C. Candidiasis of the inner leaf of the foreskin
D. Shingles
116. A man came to an urologist with E. Erythema multiforme
complains of painful urination, discharge
from urethra. The patient has been suffering 120. A 1,5-month-old child on breasfeedi-
from this condition for a week. Objectively: ng presents from birth with daily vomiting,
hyperemic urinary meatus, edema, purulent irregular liquid foamy feces, and meteori-
discharge. Microscopy of smears detected sm, which are resistant to antibacterial and
gram-negative bacteria. Specify the diagnosis: probiotic therapy; no increase of body mass
is observed. The child’s condition improved,
A. Acute gonorrheal urethritis when breastmilk was substituted with ”NAN
B. Trichomonas urethritis low lactose” formula. What pathology is it?
C. Candidal urethritis
D. Chlamydial urethritis
E. Chronic gonorrhea
117. A patient is 28 years old. He has been
suffering from mental disorder since he was
Крок 2 Medicine (англомовний варiант, iноземнi студенти) 2017 рiк 16

A. Lactase deficiency crackles in the lower segments of the lungs.


B. Intestinal lambliasis (Giardiasis) Heart sounds are weakened, the II heart
C. Infectious enteritis sound is accentuated over the pulmonary
D. Drug-induced enteritis artery. The liver is +3 cm. What complicated
E. Functional dyspepsia the clinical course of COPD in this patient?
121. A 12-year-old girl after a case of respi- A. Chronic pulmonary heart
ratory infection developed dyspnea at rest, B. Pulmonary embolism
paleness of skin. Heart rate is 110/min., BP C. Acute left ventricular failure
is 90/55 mm Hg. Heart sounds are muffled. D. Diffuse pneumosclerosis
Borders of relative heart dullness: right - the E. Community-acquired pneumonia
parasternal line, upper - the III rib, left -
1,0 cm outwards from the midclavicular line. 125. A multigravida at 39 weeks of gestation
Make the provisional diagnosis: presenting with regular labour activity for 8
hours has been delivered to a hospital; the
A. Infectious myocarditis waters broke an hour ago. She complains of
B. Functional cardiopathy headache, seeing spots. BP is 180/100 mm Hg.
C. Somatoform autonomic dysfunction Urine test results: protein - 3,3 g/l, hyaline
D. Hypertrophic cardiomyopathy cylinders. Fetal heart rate is 140/min, rhythmi-
E. Exudative pericarditis cal. Vaginal examination reveals complete
cervical dilatation, the fetal head is on the
122. A 48-year-old man complains of fati- pelvic floor, sagittal suture is in line with
gue, excessive sweating, severe skin itchi- obstetric conjugate, the occipital fontanel is
ng, undulant fever, enlarged cervical and under the pubis. What is the optimal tactics of
supraclavicular lymph nodes. Objectively: labour management?
paleness of skin and mucosa, cervical lymph
nodes are mobile, dense, elastic, walnut-sized, A. Outlet forceps
painless, not attached to the skin. Complete B. Cavity forceps
blood count: erythrocytes - 3, 0 · 1012 /l, Hb- C. Cesarean section
100 g/l, leukocytes - 14 · 109 /l, eosinophils - D. Vacuum extraction of the fetus
6%, basophils - 3%, band neutrophils - 11%, E. Conservative labour management
segmented neutrophils - 69%, lymphocytes -
126. After the contact with chemicals a plant
7, monocytes - 4%, platelets - 280 · 109 /l, ESR- worker has suddenly developed stridor, voi-
37 mm/hour. What method should be applied ce hoarseness, barking cough, progressi-
to verify the diagnosis? ng dyspnea. Objective examination reveals
A. Lymph node biopsy acrocyanosis. What is the provisional di-
B. Sternal puncture agnosis?
C. Muscle biopsy A. Laryngeal edema
D. Chest X-ray B. Laryngeal carcinoma
E. Lumbar puncture C. PATE
123. A patient after a blow to the head D. Pulmonary atelectasis
developed general symptoms of cerebral di- E. Pneumothorax
sturbance, nausea, vomiting, focal signs - 127. An 18-year-old woman complains of pain
hemi-hyperreflexia S>D, hemihyperesthesia in her lower abdomen, profuse purulent di-
on the left, marked meningeal syndrome. Nei- scharge from the vagina, temperature rise up
ther cranial X-ray nor computer tomography to 37,8o C . Anamnesis states that she had a
revealed any pathologies. What examination random sexual contact the day before the si-
method would allow making and clarification gns appeared. She was diagnosed with acute
of the diagnosis? bilateral adnexitis. On additional examinati-
A. Lumbar puncture on: leukocytes are present throughout all visi-
B. Echoencephalography on field, bacteria, diplococci with intracellular
C. Electroencephalography and extracellular position. What is the most
D. Angiography likely agent in the given case?
E. Pneumoencephalography A. Neisseria gonorrhoeae
124. A 72-year-old man complains of lower B. Escherichia coli
extremity edema, sensation of heaviness in C. Chlamydia trachomatis
the right subcostal area, dyspnea of rest. D. Trichomona vaginalis
For over 25 years he has been sufferi- E. Staphylococcus aureus
ng from COPD. Objectively: orthopnea, 128. A 58-year-old man complains of general
jugular venous distention, diffuse cyanosis, weakness, loss of 10 kg of weight within 1,5
acrocyanosis. Barrel chest is observed, months, progressive pain in the lumbar regi-
on percussion there is vesiculotympanitic on, increased blood pressure up to 220/160
(bandbox) resonance, sharply weakened vesi- mm Hg, low grade fever. Objectively: in the
cular respiration on both sides, moist crepitant
Крок 2 Medicine (англомовний варiант, iноземнi студенти) 2017 рiк 17

right hypochondrium deep palpation reveals a A. Cause of death


formation with uneven surface and low mobi- B. Manner of death
lity; veins of the spermatic cord and scrotum C. Time of death
are dilated. Blood test results: Hb- 86 g/l, D. Mode of death
ESR- 44 mm/h. Urine test results: specific E. Mechanism of death
gravity - 1020, protein - 0,99 g/l, RBC cover
the whole field of vision, WBC- 4-6 in the field 132. It is the 3rd day after the normal term
of vision. What is the provisional diagnosis? labor; the infant is rooming-in with the
mother and is on breastfeeding. Objectively:
A. Renal tumour the mother’s general condition is satisfactory.
B. Urolithiasis Temperature is 36,4o C , heart rate is 80/min.,
C. Acute pyelonephritis BP is 120/80 mm Hg. Mammary glands
D. Acute glomerulonephritis are soft and painless; lactation is moderate,
E. Nephroptosis unrestricted milk flow. The uterus is dense,
the uterine fundus is located by 3 fingers width
129. Anamnesis of a 30-year-old patient below the navel. Lochia are sanguino-serous,
includes closed thoracic injury. Lately the moderate in volume. Assess the dynamics of
patient has been suffering from increasing uterine involution:
dyspnea, sensation of heaviness in the right
subcostal area, and heart rate disturbances. A. Physiological involution
Objectively: acrocyanosis, bulging cervical B. Subinvolution
veins, ascites, edema of the lower extremiti- C. Lochiometra
es. Heart auscultation reveals muffled heart D. Pathologic involution
sounds, additional III heart sound is detected. E. Hematometra
Provisional diagnosis of constrictive peri-
carditis was made. What diagnostic technique 133. Survey radiograph of a 52-year-old
would NOT confirm the diagnosis? worker of an agglomeration plant (28-year-
long record of service, the concentration
A. US of abdomen of metal dust is 22-37 mg/m3 ) shows mi-
B. Computer tomography ldly pronounced interstitial fibrosis with di-
C. Echocardiography ffused contrasting well-defined small nodular
D. Magnetic resonance imaging shadows. The patient has no complaints.
E. Chest X-ray Pulmonary function is not compromised.
What is the provisional diagnosis?
130. A 72-year-old woman suffers from di-
abetes mellitus type 2, concomitant diseases A. Siderosis
are stage 2 hypertension and stage 2B heart B. Silicosis
failure. She takes metformin. Hypertensic cri- C. Anthraco-silicatosis
sis had occurred the day before, after whi- D. Silicatosis
ch the patient developed extreme weakness, E. Anthracosis
myalgias, thirst, dry mouth, polyuria. BP
is 140/95 mm Hg, heart rate is 98/min., 134. A 53-year-old woman complains of wei-
no edemas or smell of acetone detected. ght loss up to 10 kg within the last 2 years,
What measures should be taken to prevent liquid foul-smelling stool two times a day that
development of comatose state in the patient? poorly washes off the toilet, periodic bouts
of nausea, girdle pain in the upper abdomen.
A. Stop metformin, prescribe short-acting Objectively: pain in Gubergrits zone (on the
insulin right from navel) and at Mayo-Robson’s poi-
B. Double the dosage of metformin nt. Biochemical blood analysis: glucose - 3,2
C. Apply hypotonic solution of sodium chlori- mmol/l, bilirubin - 16,5 mcmol/l, crude protein
de - 56,4 g/l. Urine diastase/amylase - 426 g/h/l.
D. Additionally prescribe long-acting insulin D-xylose test (oral administration of 25 g of
E. Prescribe glibenclamide d-xylose) after 5 hours reveals 3 g of xylose in
131. The body of a 24-year-old woman with urine. The most likely diagnosis is:
probable signs of poisoning has been found on A. Pancreatitis. Malabsorption syndrome
the street. Forensic medical examination was B. Pseudomembranous colitis
requested by an investigator during exami- C. Nonspecific ulcerative colitis
nation of the site and the body. According D. Irritable bowel syndrome
to the Criminal Procedure Code currently in E. Chronic gastritis
force in Ukraine, forensic medical examinati-
on is required when it is necessary to determi- 135. A 15-year-old teenager has undergone
ne the: medical examination in military recruitment
center. The following was revealed: interval
systolic murmur at the cardiac apex, accent
of the II heart sound over the pulmonary
artery, tachycardia. What additional exami-
nation method will be the most informative
Крок 2 Medicine (англомовний варiант, iноземнi студенти) 2017 рiк 18

for determining diagnosis? A. Reactive arthritis


B. Rheumatoid arthritis
A. Echocardiography C. Seasonal pollinosis
B. Electrocardiography D. Bacterial nonspecific urethral conjunctivitis
C. X-ray E. Upper respiratory tract infection (URTI)
D. Phonocardiography that affects conjunctiva and joints
E. Rheography
140. A 19-year-old woman complains of
136. A 64-year-old patient has been hospitali- severe pain in the axillary crease. Conditi-
zed with complaints of progressive jaundice on onset occurred a week ago after her swi-
that developed over 3 weeks without pain mming in a cold river and epilation. The next
syndrome and is accompanied by general day a painful ”boil” appeared. The ”boil” was
weakness and loss of appetite. Objectively: increasing in size every day and became a
temperature is 36,8o C , heart rate is 78/min, plum-sized tumor. Upon examination there
abdomen is soft and painless, peritoneum irri- are nodular conical growths joined together
tation symptoms are not detected, palpati- detected, the skin covering them is bluish-red
on reveals sharply enlarged tense gallbladder. in color. Some nodules have fistulous openi-
What disease can be characterised by these ngs producing thick purulent mass. Body
symptoms? temperature is 38, 5o C , general malaise. What
is the most likely diagnosis?
A. Cancer of pancreas head
B. Duodenal ulcer A. Hydradenitis
C. Acute cholecystitis B. Carbuncle
D. Chronic cholecystitis C. Cutaneous tuberculosis
E. Cholecystitis caused by lambliasis D. Necrotizing ulcerative trichophytosis
E. Pyoderma chancriformis
137. A 6-year-old girl came to a general practi-
tioner with her mother. The child complains 141. A woman complains of weight gain,
of burning pain and itching in her external chills, edema, xeroderma, somnolence, diffi-
genitalia. The girl was taking antibiotics the culties with focusing. Objectively: height is
day before due to her suffering from acute 165 cm; weight is 90 kg; body proportions
bronchitis. On examination: external genitalia are of female type, to - 35,8o C , heart rate -
are swollen, hyperemic, there is white deposit 58/min, BP- 105/60 mm Hg. Heart sounds
accumulated in the folds. The most likely di- are weakened, bradycardia is observed. Other
agnosis is: internal organs have no alterations. Thyroid
gland cannot be palpated. Milk secretion
A. Candidal vulvovaginitis from mammary glands is observed. Hormone
B. Trichomoniasis test revealed increased levels of thyroid-
C. Nonspecific vulvitis stimulating hormone (TSH) and prolactin,
D. Helminthic invasion and decreased level of thyroxine (4 ). What
E. Herpes vulvitis is the cause of obesity?
138. A 37-year-old patient complains of pain A. Primary hypothyroidism
in the spinal column, reduced mobility. The B. Secondary hypothyroidism
condition persists for 7 years. ”Sway back” is C. Prolactinoma
observed, there is no movement in all spi- D. Hypopituitarism
nal regions. On X-ray: ”bamboo spine” is E. Adiposogenital dystrophy
detected. What is the most likely diagnosis?
142. ECG revealed the following in a 10-
A. Ankylosing spondylitis year-old child: sharp acceleration of the heart
B. Osteochondrosis rate - 240/min., P wave overlaps with T wave
C. Spondylitis deformans and deforms it, moderate lengthening of PQ
D. Tuberculous spondylitis interval, QRS complex is without alterations.
E. Spondylolisthesis What pathology does this child have?
139. A 25-year-old patient is not married and A. Paroxysmal atrial tachycardia
has sexual relations with several partners. B. Atrial hypertrophy
During the last 3 months he noticed a small C. Ventricular hypertrophy
amount of mucoserous discharge from the D. WPW syndrome
urethra. Subjectively: periodical itching or E. Extrasystole
burning pain in the urethra. Two months ago
pain in the knee joint developed. Possibility 143. A 54-year-old patient complains of
of trauma or exposure to cold is denied by the weakness, jaundice, itching skin. Disease
patient. During the last week eye discomfort onset was 1,5 months ago: fever up to 39o C
is noted - lacrimation and itching. What provi- appeared at first, with progressive jaundice
sional diagnosis can be made? developed 2 weeks later. On hospitalizati-
on jaundice was severely progressed. Liver
cannot be palpated. Gallbladder is enlarged
Крок 2 Medicine (англомовний варiант, iноземнi студенти) 2017 рiк 19

and painless. Blood bilirubin is 190 mcmol/l analyses periodically revealed leukocyturia.
(accounting for direct bilirubin). Stool is The child has undergone no further treatment.
acholic. What is the most likely jaundice On examination: increased BP up to 150/100
genesis in this patient? mm Hg. Ultrasound investigation revealed si-
gnificant reduction of the right kidney. What
A. Mechanical jaundice process is leading in arterial hypertension
B. Hepatocellular jaundice pathogenesis in this case?
C. Hemolytic jaundice
D. Caroli syndrome A. Hyperactivity of renin-angiotensin system
E. Gilbert’s syndrome B. Disturbance of water-electrolytic balance
C. Disturbance of renal circulation
144. During hemotransfusion the patient D. Hypersympathicotonia
developed nausea, tremor, lumbar and E. Increased cortisol level
retrosternal pain. On examination the skin
is hyperemic, later developed pallor; the pati- 148. A 3-month-old child presents with
ent presents with hyperhidrosis, labored respi- saffron-yellow coloring of the skin, sclera,
ration, pulse is 110/min., BP is 70/40 mm Hg. and mucous membranes. The abdomen is
Urine is black colored. What complication enlarged, hepatomegaly and splenomegaly
developed in the patient? are observed. In blood there is conjugated
bilirubin-induced hyperbilirubinemia. On
A. Posttransfusion shock intravenous cholangiocholecystography:
B. Acute renal failure opacified bile is discharged into the intesti-
C. Pulmonary embolism ne. Transaminase activity is normal. What is
D. Anaphylactic shock the most likely diagnosis?
E. Hypotonic crisis
A. Biliary atresia
145. A 22-year-old man suddenly developed B. Physiologic jaundice
extreme weakness, nausea, vomiting with C. Hemolytic disease of newborn
traces of blood. The patient is known to D. Crigler-Najjar syndrome
suffer from peptiv ulcer disease of duodenum E. Congenital hepatitis
and hemophilia A. Objectively: heart rate -
102/min., BP- 100/60 mm Hg. Complete blood 149. A 51-year-old woman complains of
count: erythrocytes - 3, 2 · 1012 /l, Hb- 98 g/l, headache, trembling, paresthesiae, palpi-
color index - 0,92, leukocytes - 7, 4 · 109 /l, tations, increased blood pressure up to
280/160 mm Hg. The day before she experi-
platelets - 240 · 109 /l, ESR- 11 mm/hour. What enced exhausting headache, vascular pulsati-
measure would most effectively decrease on, palpitations, asphyxia, stomachache,
hemorrhaging in this case? unbearable fear of coming death. The pati-
A. Cryoprecipitate ent paled and broke out in cold sweat. In
B. Aminocapronic acid urine there is increased content of vani-
C. Native plasma llylmandelic acid. What disease causes such
D. Direct transfusion of donor blood clinical presentation in the patient?
E. Platelet concentrate transfusion A. Pheochromocytoma
146. A 22-year-old woman complains of B. Conn’s syndrome (primary
amenorrhea for 8 months. Anamnesis states hyperaldosteronism)
that menarche occured at the age of 12,5. Si- C. Cushing’s syndrome
nce the age of 18 the patient has a history of D. Primary hypertension
irregular menstruation. The patient is nulli- E. Cushing’s disease
gravida. The mammary glands are developed
properly, nipples discharge drops of milk 150. A 52-year-old patient suffers from
when pressed. Hormone test: prolactin level marked dyspnea during physical exertion,
is 2 times higher than normal. CT reveals a non-productive cough. The patient’s condi-
bulky formation with diameter of 4 mm in tion has been persisting for 8 months. The
the region of sella. What is the most likely di- patient has been a smoker for 30 years. In
agnosis? the lungs there are cellophane-type crackles
auscultated on both sides. Respiration rate is
A. Pituitary tumour 26/min., oxygen saturation of blood is 92%.
B. Lactational amenorrhea On spirometry: moderate restrictive-type di-
C. Stein-Leventhal syndrome (polycystic ovary sturbance of external respiration. What is the
syndrome) most likely diagnosis?
D. Sheehan’s syndrome (postpartum hypopi-
tuitarism)
E. Cushing’s disease
147. A 13-year-old girl complains of fatigabili-
ty, frequent headaches, cardialgia. Eight years
ago she had a case of pyelonephritis. Urine
Крок 2 Medicine (англомовний варiант, iноземнi студенти) 2017 рiк 20

A. Idiopathic fibrosing alveolitis A. Lymphogranulomatosis (Hodgkin’s


B. Chronic obstructive pulmonary disease lymphoma)
(COPD) B. Lymph node tuberculosis
C. Chronic bronchitis C. Lymphoreticulosarcoma
D. Community-acquired pneumonia D. Cancer metastases to the lymph nodes
E. Sarcoidosis E. Macofollicular reticulosis
151. A 26-year-old patient with affective bi- 155. An 9-year-old child was hospitalized
polar disorder has developed a condition for fever up to 39,8o C , inertness, moderate
manifested by mood improvement, behavi- headache, vomiting. Examination revealed
oural and sexual hyperactivity, verbosity, acti- meningeal symptoms. Lumbar puncture was
ve body language, reduced need for sleep. performed. The obtained fluid was characteri-
Which of the following drugs would be most sed by increased opening pressure, was
effective in this case? transparent, with the cell count of 450 cells per
1 mcL (mainly lymphocytes - 90%), glucose
A. Neuroleptics with sedative effect level of 3,6 mmol/l. What agent could have
B. Antidepressants with activating effect caused the disease in the child?
C. Neuroleptics with activating effect
D. Tranquilizers A. Enterovirus
E. Antidepressants with sedative effect B. Neisseria meningitidis
C. Mycobacterium tuberculosis
152. A 19-year-old patient complains of D. Staphylococcus aureus
dyspnea on exertion. He often has bronchitis E. Streptococcus pneumoniae
and pneumonia. Since childhood the patient
presents with cardiac murmur. Auscultation 156. A 25-year-old woman has a self-detected
revealed splitting of the II sound above the tumor in the upper outer quadrant of her ri-
pulmonary artery, systolic murmur in the 3rd ght breast. On palpation there is a painless
intercostal space at the left sternal border. firm mobile lump up to 2 cm in diameter, peri-
ECG detected right bundle branch block. pheral lymph nodes are without alterations. In
What is the provisional diagnosis? the upper outer quadrant of the right breast
ultrasound revealed a massive neoplasm with
A. Atrial septal defect increased echogenicity sized 21x18 mm. What
B. Open ductus arteriosus is the most likely diagnosis?
C. Aortarctia
D. Aortic stenosis A. Fibroadenoma
E. Mitral insufficiency B. Lactocele
C. Diffuse mastopathy
153. A 30-year-old patient was in a car acci- D. Mammary cancer
dent. He is unconscious, pale, has thready E. Mastitis
pulse. In the middle third of the right thigh
there is an extensive laceration with ongoi- 157. A 68-year-old man complains of inabili-
ng profuse external arterial bleeding. What ty to urinate for a day. On attempt of urinary
urgent actions must be taken to save the life bladder catheterization there was detected a
of the patient? rough stricture in the membranous portion of
the urethra. What first aid tactics should be
A. Tourniquet above the wound of the right applied in this case?
thigh
B. Tourniquet below the wound of the right A. Troacar cystostomy
thigh B. Adenomectomy
C. Artificial lung ventilation C. Optical internal urethrotomy
D. Precordial thump D. α-adrenoblockers
E. Application of plaster bar E. Urinary bladder catheterization
154. A 25-year-old patient has been admi- 158. Heart X-ray of a 31-year-old man has
tted to the hospital with the following revealed the following: with tightly filled
problems: weakness, sweating, itching, wei- opacified esophagus there is a marginal filli-
ght loss, enlarged submandibular, cervical, ng defect in its middle third on the posterior
axillary, inguinal lymph nodes. Objectively: wall; the defect is 1,8x1,3 cm in size with clear
hepatomegaly. Lymph node biopsy revealed oval border. Mucosal folds are retained and
giant Berezovsky-Reed-Sternberg cells, envelop the defect; wall peristalsis and elasti-
polymorphocellular granuloma composed of city are not affected. There are no complai-
lymphocytes, reticular cells, neutrophils, eosi- nts regarding the condition of the patient’s
nophils, fibrous tissue, and plasma cells. What alimentary canal. Make the provisional di-
is the most likely diagnosis? agnosis:
Крок 2 Medicine (англомовний варiант, iноземнi студенти) 2017 рiк 21

A. Esophageal tumor general fatigue, low-grade fever persisting


B. Achalasia cardiae for 4 months, lumbar pain, and dysuria.
C. Esophageal burns Anamnesis includes frequent acute respi-
D. Diverticulum ratory diseases, overexposure to cold, low-
E. Barrett esophagus calorie diet, a case of pulmonary tuberculosis
in childhood. Clinical urine analysis: pH-
159. A patient with signs of general 4,8, leukocyturia, hematuria. Complete blood
overexposure to cold presenting with local count: leukocytosis, lymphocytosis, increased
frostbites of fingers has been delivered into ESR. Urography concludes: dilatation of
an admission room. Objectively: conscious, renal pelvis and calyceal system of both ki-
inert, speech is slow, the skin of the face is dneys, foci of calcification in the projection of
cold, body temperature is 34o C , heart rate is right kidney parenchyma. What is the most
68/min. What would be the actions of a doctor likely diagnosis?
on call?
A. Nephrotuberculosis
A. Hospitalize the patient to the surgical B. Right renal cyst
department C. Right renal carcinoma
B. Hospitalize the patient to the therapeutics D. Acute glomerulonephritis
department E. Chronic pyelonephritis
C. Hospitalize the patient to the traumatology
department 164. A full-term newborn (born with the body
D. Let the patient go home weight of 3900 g at gestational age of 39
E. Refer to a family doctor on the next day weeks) on the first day of his life developed
160. A 22-day-old infant developed respiratory disturbances: dyspnea, arrhythmic
subcutaneous red nodes from 1,0 to 1,5 cm in respiration, cyanosis attacks. On examination
size on the scalp; later the nodes suppurated. there is paradoxical respiration observed and
Temperature increased up to 37,7o C , intoxi- left side of the chest lags behind in the act of
cation symptoms appeared, regional lymph breathing. On auscultation the respiration is
nodes enlarged. Complete blood count: weakened in the lungs on the left. Neurologi-
anemia, leukocytosis, neutrocytosis, increased st diagnosed the patient with left-sided Erb-
ESR. What diagnosis will you make? Duchenne palsy. Complete blood count shows
no changes. What is the most likely diagnosis?
A. Pseudofurunculosis
B. Pemphigus A. Left-sided diaphragm paresis
C. Vesiculopustulosis B. Congenital pneumonia
D. Scalp phlegmon C. Left-sided pneumothorax
E. - D. Respiratory distress syndrome
E. Transient tachypnea of the newborn
161. During examination of a healthy infant,
the child takes a toy into his hands, turns 165. A 12-year-old child had three attacks of
from the back to the side; when lying on the acute rheumatic fever accompanied by cardi-
stomach he can firmly prop himself up on his tis. Examination revealed the symptoms of
forearms; the child laughs and makes joyful chronic tonsillitis, mitral insufficiency, and
exclamations. The age of the child is: carious teeth. What is the optimal method
of secondary prophylaxis?
A. 4 months
B. 2 months A. Year-round bicillin prophylaxis until the age
C. 5 months of 25
D. 1 month B. Course of cardiotrophic drugs twice a year
E. 6 months C. Year-round bicillin prophylaxis for 3 years
D. Tonsillectomy
162. After significant physical exertion a 66- E. Oral cavity sanitation
year-old man with deep vein thrombosis of
the extremities developed shortness of breath, 166. A 24-year-old patient had been delivered
intense pain in the chest on the left, marked to a thoracic department with a chest injury,
palpitations. The patient’s condition is grave, fracture of the IV, V, VI ribs on the right.
his face is cyanotic, the cervical veins are Plan radiography showed the fluid level in
swollen, BP is 60/40 mm Hg. What investi- the pleural cavity to be reaching the III rib
gation method would be the most advisable in on the right. Puncture contained blood clots.
this case? What is the optimal treatment tactics?

A. Selective angiopneumography A. Emergency thoracotomy


B. Chest X-ray B. Pleural puncture
C. Echocardiography C. Thoracentesis and thoracostomy
D. Magnetic resonance imaging of the chest D. Hemostatic therapy
E. Fiber-optic bronchoscopy E. Medical thoracoscopy

163. A 32-year-old woman complains of 167. A 3-year-old child has been delivered to
Крок 2 Medicine (англомовний варiант, iноземнi студенти) 2017 рiк 22

a hospital with complaints of pain in the legs, 171. Examination of a Rh-negative pregnant
fever, loss of appetite. Objectively: pale skin woman at 32 weeks of gestation revealed
and mucosa, hemorrhagic rash. Lymph nodes a four-time rise of Rh-antibody titer within
are enlarged, painless, dense and elastic, not 2 last weeks; the titer is 1:64. The first two
matted together. Bones, joints, and abdomen pregnancies resulted in antenatal fetal death
are painful. The liver and spleen are enlarged. due to hemolytic disease. What is the optimal
Hemogram: Hb- 88 g/l, color index - 1,3, tactics of pregnancy management?
platelets - 80 · 109 /l, leukocytes - 25, 8 · 109 /l,
lymphoblasts - 70%, ESR- 52 mm/hour. Make A. Early delivery
the provisional diagnosis: B. Delivery at 37 weeks of gestation
C. Screening for Rh-antibodies in 2 weeks and
A. Acute leukemia urgent delivery in case of further increase of
B. Thrombocytopenic purpura antibody titer
C. Acute rheumatic fever D. Introduction of anti-Rh (D)
D. Infectious mononucleosis immunoglobulin
E. Hemorrhagic vasculitis (Henoch-Schonlein E. Ultrasound for signs of hemolytic disease of
purpura) the fetus
168. A 60-year-old man has a diet consisting of 172. During last several weeks an 11-year-
unvaried food staples: mostly cereals, potato, old girl has been complaining of dyspnea and
pasta; few vegetables and little fats (especially edema of shins and feet after physical exerci-
animal fats). During medical examination he se. After a long rest or sleep through the night
complains of deterioration of his twilight visi- her edemas diminish significantly. On clini-
on. This condition can be caused by lack of: cal examination there are enlarged liver and
rasping systolic murmur over the cardiac area.
A. Retinol Blood and urine analyses are without changes.
B. Amino acids What is the most likely cause of the child’s
C. Fats edema?
D. Calcium
E. Carbohydrates A. Heart failure
B. Angioneurotic edema
169. A 14-year-old girl came to a general C. Acute pyelonephritis
practitioner with complaints of weakness, D. Hepatocirrhosis
loss of appetite, headache, rapid fatigabili- E. Nephrotic syndrome
ty. Her last menstruation was profuse and
lasted for 14 days after previous delay of 2 173. A 74-year-old man visited an urologist
months. Objectively: the skin is pale, heart with complaints of pain above the pubis and
rate is 90/min., BP is 110/70 mm Hg, Hb is inability to urinate for 8 hours. At home he
88 g/l. Rectal examination: the uterus and its had taken antispasmodics and had a warm
appendages are without changes, no discharge bath but no improvement occurred. Objecti-
from the genital tracts. What complication vely: the abdomen is soft and painful above
occurred in the patient? the pubis; dullness of percussion sound is
observed above the pubis. Murphy’s punch si-
A. Posthemorrhagic anemia gn (costovertebral angle tenderness) is negati-
B. Somatoform autonomic dysfunction of ve on both sides. What condition does the
hypotonic type patient have?
C. Migraine
D. Gastritis A. Acute urinary retention
E. Dysmenorrhea B. Paradoxal ischuria
C. Chronic urinary retention
170. A postpartum woman on the 12th day D. Anuria
after the normal delivery complains of pain E. Oliguria
localized in her left gastrocnemius muscle.
Body temperature is 37,2o C ; pulse is 85/min, 174. During preventive examination a 58-year-
rhythmic; blood pressure is 128/80 mm Hg. old man on chest X-ray presents with multi-
Mammary glands are soft and painless. The ple globular pale shadows 3 cm in diameter
uterus is behind the pubis. The left leg in the within parenchyma of the both lungs. Exami-
area of gastrocnemius muscle is by 3 cm larger nation in the oncologic hospital: the primary
than the right leg in the diameter. Internal focus is not found; transbronchial biopsy wi-
organs present no pathologies. What compli- th cytologic investigation detected cells of
cation can be suspected? glandular neoplasm. What tactics should the
physician choose?
A. Deep vein thrombosis of the shin
B. Iliofemoral thrombosis
C. Varicose veins of lower extremities
D. Endometritis
E. Myositis
Крок 2 Medicine (англомовний варiант, iноземнi студенти) 2017 рiк 23

A. Polychemotherapy courses on on the right, which developed late in


B. Exploratory laparotomy the night, and impaired walking. Objecti-
C. Exploratory thoracotomy vely: the metatarsophalangeal articulation
D. Laparoscopy is swollen, hyperemic, hot to touch, pai-
E. Symptomatic treatment at home nful on movement. In blood: erythrocytes -
175. A 36-year-old man developed a disease 5, 1·1012 /l, Нb- 155 g/l, leukocytes- 13, 0·109 /l,
with acute onset 6 hours ago. The patient ESR- 50 mm/hour, CRP- 46 mg/dl, uric
presents with pain in the epigastric, ileocecal, acid - 720 mcmol/l. X-ray of feet articulati-
and paraumbilical areas, vomiting, weakness, ons: osteoporosis, narrowing of interarti-
nausea, and body temperature of 38,5o C . cular spaces, numerous punched-out erosions.
Stool is liquid, profuse, frequent, retains fecal Make the preliminary diagnosis:
nature, foul-smelling, frothy, colored dark A. Gout
green. The stomach is moderately distended B. Osteoarthritis
and painful on palpation. The patient attri- C. Reactive arthritis
butes his disease to eating raw chicken eggs D. Rheumatoid arthritis
one day before the clinical signs of the disease E. Psoriatic arthritis
appeared. What is the most likely diagnosis?
179. A woman in her early- to mid-thirties
A. Salmonellosis has lost her consciousness 3-5 minutes ago.
B. Shigellosis On examination: the skin is pale, no pulse
C. Typhoid fever over the carotid arteries, no spontaneous
D. Cholera respiration, pupils are dilated; the patient is
E. Food toxicoinfection nonresponsive, presents with atony. The pati-
176. A 55-year-old woman came to a ent’s condition can be determined as:
gynecologist with complaints of leukorrhea A. Apparent death
and bloody discharge from the vagina after B. Natural death
5 years of menopause. Anamnesis states C. Syncope
no pregnancies. Bimanual examination: the D. Brain death
uterus and uterine appendages are without E. Coma
changes. During diagnostic curettage of the
uterine cavity the physician scraped off 180. A 32-year-old woman complains of body
enchephaloid matter. What is the most likely weight loss despite her increased appetite,
diagnosis in this case? nervousness, and tremor of the extremities.
Objectively: the skin is moist; the thyroid
A. Endometrial carcinoma gland is diffusely enlarged, painless, soft, and
B. Adenomyosis mobile. Blood test: increased level of T3,
C. Subserous uterine myoma T4, and thyroid-stimulating hormone (THS).
D. Cervical carcinoma What is the most likely diagnosis?
E. Ovarian carcinoma
A. Diffuse toxic goiter
177. A 10-year-old boy is delivered into a B. Thyroid carcinoma
polytrauma unit after he received a blunt C. Autoimmune (Hashimoto’s) thyroiditis
trauma of the thorax, having fallen from D. Thyroid adenoma
the bicycle. Upon hospitalization his blood E. Diffuse nontoxic goiter
pressure is 110/80 mm Hg, heart rate is 96/min.
Chest X-ray is noncontributive to the di- 181. A 57-year-old patient complains of
agnosis. Echocardiogram shows free liquid sensation of dryness and pain during
in the pericardial cavity, in the amount of up swallowing, frequent unbearable cough, the
to 100 ml. In an hour after the hospitalization voice is hoarse. Disease onset was abrupt. On
the patient started to develop increasing si- laryngoscopy: laryngeal mucosa is hyperemic,
gns of heart failure: jugular venous distention, vocal folds are swollen, laryngeal lumen
decreased blood pressure down to 90/70 mm contains viscous secretion. What diagnosis is
Hg, tachycardia up to 120/min. On auscultati- it?
on muffled heart sounds. What would be the
primary tactics of a physician? A. Acute laryngitis
B. Acute stenosing laryngotracheitis
A. Pericardiocentesis C. Bronchial asthma
B. Cardiac glycosides intravenously D. Flegmonous laryngitis
C. Constant oxygenotherapy E. Laryngeal diphtheria
D. Diuretics intravenously
E. Antibiotics intravenously 182. A 24-year-old pregnant woman on her
37th week of pregnancy has been delivered to
178. After the celebratory feast that took a maternity obstetric service with complaints
place the day before, a 35-year-old man was of weak fetal movements. Fetal heartbeats are
hospitalized with complaints of marked pain 95/min. On vaginal examination the uterine
within the I metatarsophalangeal articulati- cervix is tilted backwards, 2 cm long, external
Крок 2 Medicine (англомовний варiант, iноземнi студенти) 2017 рiк 24

orifice allows inserting a fingertip. Biophysi- A. Total protein content in the pleural fluid
cal profile of the fetus equals 4 points. What below 25 g/l
tactics of pregnancy management should be B. Presence of atypical cells
chosen? C. Total protein content in the pleural fluid
above 30 g/l
A. Urgent delivery via a cesarean section D. Specific gravity exceeding 1015
B. Treatment of placental dysfunction and E. Positive Rivalta’s test
repeated analysis of the fetal biophysical
profile on the next day 186. A 28-year-old woman complains of gi-
C. Doppler measurement of blood velocity in rdle pain in her epigastric and left subcostal
the umbilical artery areas with irradiation to the back, nausea,
D. Urgent preparation of the uterine cervix for and vomiting without relief. On examinati-
delivery on a surgeon observes stomach distension
E. Treatment of fetal distress, if ineffective, and meteorism. There are positive Mondor’s,
then elective cesarean section on the next day Mayo-Robson’s, and Cullen’s symptoms.
What is the most likely diagnosis?
183. During regular preventive gynecologi-
cal examination a 30-year-old woman was A. Acute pancreatitis
detected to have dark blue punctulated B. Acute cholecystitis
”perforations” on the vaginal portion of C. Acute intestinal obstruction
the uterine cervix. The doctor suspects D. Aortic dissecting aneurysm
endometriosis of the vaginal portion of the E. Splenic infarction
uterine cervix. What investigation method
would be most informative for diagnosis 187. A 36-year-old woman complains of
confirmation? pain in her lumbar area, which irradiates to
her lower right limb and increases during
A. Colposcopy, target biopsy of the cervix movements, and sensation of numbness in
B. US of small pelvis her limb. Objectively: palpation of the shin
C. Hysteroscopy and thigh muscles is painful, positive stretch
D. Curettage of the uterine cavity symptom on the right. MRI scan: herniati-
E. Hormone testing on of intervertebral disk L5-S1 4 mm in size.
What is the most likely diagnosis?
184. A 26-year-old woman came to a
gynecologist for a regular check-up. She has A. Vertebrogenic lumbago
no complaints. Per vaginum: the uterus lies in B. Vertebrogenic radicular syndrome of L5-S1
anteflexion, not enlarged, dense, mobile, pai- on the right
nless. On the left from the uterus in the area of C. Endarteritis of lower extremities
uterine appendages there is a mobile painless D. Spinal stroke
outgrowth that can be moved independently E. Acute myelitis
from the uterus. On the right the appendages
cannot be detected. What additional investi- 188. During routine medical examinati-
gation would be informative for diagnosis on a 35-year-old woman presents with
clarification? enlarged cervical and mediastinal lymph
nodes. Her overall health is satisfactory.
A. US of lesser pelvis ESR is 30 mm/hour. Cervical node biopsy
B. Metrosalpingography was performed. In the specimen there are
C. Examination for urogenital infection granulomas composed of epithelial and gi-
D. Colposcopy ant cells, no caseous necrosis detected. What
E. Colonoscopy is the most likely diagnosis?

185. A 57-year-old patient complains of A. Sarcoidosis


dyspnea at rest. The patient presents with B. Lymphogranulomatosis
orthopnea, acrocyanosis, bulging cervical vei- C. Infectious mononucleosis
ns. On percussion: dull sound over the lower D. Nonspecific lymphadenitis
lung segments; on auscultation: no respiratory E. Lymph node tuberculosis
murmurs. Heart rate is 92/min. Right-sided
cardiac dilatation is observed. The liver is 189. A 39-year-old woman complains of vi-
enlarged by 7 cm. Shins are swollen. Pleural olent pain in her left lumbar area with irradi-
effusion is suspected. What indicator would ation to the right iliac area. Several years
confirm the presence of transudate in this ago she was diagnosed with cholelithiasis
case? and urolithiasis. The patient’s condition is
moderately severe, the skin is dry. Ortner’s
symptom is negative; costovertebral angle
tenderness is observed on the right. The most
reasonable treatment tactics would be:
Крок 2 Medicine (англомовний варiант, iноземнi студенти) 2017 рiк 25

A. Spasmolytics and analgesics 194. A 32-year-old woman complains of epi-


B. Urgent hemodialysis sodes of intense fear that occur without visible
C. Peritoneal dialysis cause and last for 10-20 minutes; the episodes
D. Laparoscopic cholecystectomy are characterized by rapid pulse, sweating,
E. Antibiotics labored breathing, and vertigo. Specify the li-
kely diagnosis:
190. An 18-year-old patient complains of skin
rash. The patient has been suffering from this A. Panic disorder
condition for 5 years. The first instance of this B. Paranoid syndrome
disease occurred after a car accident. Objecti- C. Manic syndrome
vely: the patient presents with papular rash D. Simple schizophrenia
covered in silvery scales, ”thimble” symptom E. Claustrophobia
(small pits on the nails), affected joints. What
is the most likely diagnosis? 195. A resuscitation unit received a 46-year-
old woman, who has been suffering from
A. Psoriasis diabetes mellitus type 1 for approximately
B. Panaritium 30 years. Objectively: the skin is pale, heart
C. Onychomycosis sounds are weakened, BP is 170/100 mm
D. Lupus erythematosus Hg, lower limbs are markedly swollen. Blood
E. Rheumatism creatinine - 1125 mcmol/l, urea - 49,6 mmol/l,
potassium - 6.3 mmol/l, glucose - 7,6 mmol/l,
191. A woman complains of frequent, li- glomerular filtration rate - 5 ml/min. What
quid stool (up to 9-10 times per day) with treatment is indicated for the patient in the
mucus and blood admixtures, dull pain in the first place?
hypogastrium, weight loss of 4 kg within the
last year. Objectively: malnutrition, dry skin, A. Hemodialysis
low turgor, aphthous stomatitis. The stomach B. Kidney transplantation
is soft, the sigmoid colon is spastic and painful C. Hemofiltration
on palpation. Occult blood test is positive. Fi- D. Enterosorption
brocolonoscopy: edema, hyperemia, mucosal E. Conservative detoxification therapy
granulation, pseudopolyps, small ulcers with
irregular edges. Make the diagnosis: 196. A woman is on the 32nd week of her
second pregnancy. She complains of fever,
A. Nonspecific ulcerative colitis chills, nausea, vomiting, lumbar pain, and
B. Chronic enterocolitis dysuria. Costovertebral angle tenderness is
C. Colon cancer present on both sides. Urine analysis: pyuria,
D. Irritable bowel syndrome bacteriuria. Blood test: leukocytosis. What is
E. Crohn’s disease (regional enteritis) the most likely diagnosis?
192. A 48-year-old woman has been hospitali- A. Gestational pyelonephritis
zed due to development of tachysystolic atrial B. Cystitis
fibrillation. She has lost 5 kg of body weight C. Pyelitis
within 2 months. On palpation there is a node D. Glomerulonephritis
in the left lobe of the thyroid gland. What E. Latent bacteriuria
pathology resulted in the development of this
condition? 197. A 19-year-old young man complains of
cough with expectoration of purulent sputum
A. Toxic nodular goiter in amount of 100 ml per day, hemoptysis,
B. Aterosclerotic cardiosclerosis dyspnea, increased body temperature up
C. Chronic thyroiditis to 37,8oC , general weakness, weight loss.
D. Nontoxic nodular goiter The patient’s condition lasts for 4 years.
E. Autoimmune thyroiditis Exacerbations occur 2-3 times a year.
The patient presents with malnutrition,
193. On the 9th day after childbirth the pale skin, cyanosis of the lips, drumstick
obstetric patient developed high fever up (clubbed) fingers. Tympanic percussion sound
to 38o C . She complains of pain in the right in the lungs, weakened respiration, vari-
mammary gland. On examination a sharply ous numerous moist crackles in the lower
painful infiltrate can be palpated in the right pulmonary segments on the left can be
mammary gland, the skin over the infiltrate observed. In blood: erythrocytes - 3, 2 · 1012 /l,
is red, subareolar area and nipple are swollen
and painful. What is your diagnosis? leukocytes - 8, 4 · 109 /l, ESR- 56 mm/hour.
On X-ray: lung fields are emphysematous, the
A. Abscess of the right mammary gland left pulmonary root is deformed and dilated.
B. Mastopathy What is the most likely diagnosis?
C. Cancer of the right mammary gland
D. Serous mastitis
E. Fibrous cystic degeneration of the right
mammary gland
Крок 2 Medicine (англомовний варiант, iноземнi студенти) 2017 рiк 26

A. Multiple bronchiectasis of the left lung her son with violence. She had opened her
B. Chronic left-sided pneumonia window (on the 8th floor) and tried to jump
C. Chronic abscess of the left lung out, resisted the people, who were holding her
D. Left-sided pulmonary cystic dysplasia back. What drugs should be administered to
E. Suppuration of the cyst in the left lung terminate such condition of the patient?
198. A 37-year-old man working as a A. Antipsychotics
typesetter in a print shop complains of B. Antidepressants
rapid fatigability, paroxysmal attacks of C. Tranquilizers
stomachache, weak drooping hands. Exami- D. Nootropic agents
nation of neurological status revealed E. Antihypertensive drugs
hypotrophy of the forearm muscles.
Carporadial reflexes are sharply weakened. 200. A 43-year-old man, who has been abusi-
Sensitivity is not disturbed. Gums present ng alcohol and suffering from pulmonary
with dark blue border. What neurological tuberculosis, in the course of two weeks
pathology is it? gradually developed general weakness,
headache, diplopia, vomiting. Objectively:
A. Lead polyneuropathy ptosis on the left, anisocoria S>D, exotropia
B. Guillain-Barre syndrome (postinfectious of the left eye, neck stiffness; Kernig’s
polyneuritis) and Brudzinski’s signs are positive. In
C. Shingles cerebrospinal fluid: lymphocytic pleocytosis,
D. Ulnar neuropathy low glucose, precipitation of cerebrospinal
E. Brachial plexitis fluid resulted in production of fibrin film.
What is the most likely diagnosis?
199. A 54-year-old woman takes anti-
hypertensive drugs for hypertension. Havi- A. Tuberculous meningitis
ng discovered that her son was arrested and B. Subarachnoid hemorrhage
is under investigation, became agitated and C. Brainstem encephalitis
extremely anxious. She lost her orientation D. Acute myelitis
in place, stopped recognizing her relatives, E. Basal arachnoiditis
started hearing ”voices” threatening her and
INSTRUCTIONAL BOOK

Testing Board

TEST ITEMS FOR LICENSING EXAMINATION: KROK 2. MEDICINE.

Kyiv. Testing Board.


(English language).

Approved to print 24.04/№217. Paper size 60х84 1/8


Offset paper. Typeface. Times New Roman Cyr. Offset print.
Conditional print pages 24. Accounting publishing pages 28.
Issue. 2610 copies
List of abbreviations
List of abbreviations
ACTH Adrenocorticotrophic hormone
ADP Adenosine diphosphate
ALT, ALAT Alanin aminotranspherase
ALV Artificial lung ventilation
AP Arterial (blood) pressure
AST, AspAT Aspartat aminotranspherase
ATP Adenosine triphosphate
ВСG Bacillus Calmette-Guérin
BP Blood (arterial) pressure
CK Creatine kinase
CNS Central nervous system
СоА Coenzyme A
CRP C-reactive protein
CT Computed tomography
COX Cyclooxygenase
DIC Disseminated intravascular coagulation
DTP (DPT) Diphtheria-tetanus-pertussis vaccine
EDTA Ethylenediamine tetra-acetic acid
ELISA Enzyme-linked immunosorbent assay
ENT Ear, nose, and throat (as a department in a hospital)
EPR (ER) Endoplasmic reticulum
ESR Erythrocyte sedimentation rate
EV Enterovirus
FC Functional class
GABA Gamma aminobutyric acid
Hct, Ht Hematocrit
HDL High-density lipoproteins
HR Heart rate
IDL Intermediate-density lipoproteins
IU International unit
IUPAC International Union of Pure and Applied Chemistry
LDH Lactate dehydrogenase
LDL Low-density lipoproteins
LOX Lipoxygenase
MAO Monoamine oxidase
MRI Magnetic resonance imaging
NSAID Nonsteroidal anti-inflammatory drug
PE (PATE) Pulmonary embolism (Pulmonary artery thromboembolism)
PSA Prostate-specific antigen
RBC Red blood count
RR Respiratory rate
SES Sanitary-and-epidemiologic station
STD Sexually transmitted disease
STI Sexually transmitted infection
T/l Trillion/liter
TABT Typhoid-paratyphoid A and B plus tetanus toxoid vaccine
TMJ Temporomandibular joint
TSH Thyroid-stimulating hormone
TU Tuberculin unit
URTI Upper respiratory tract infection
V/f Vision field
VLDL Very-low-density lipoproteins
WBC White blood count
WPW Wolff-Parkinson-White syndrome

Potrebbero piacerti anche